arterial blood gases made easy - i. hennessey, a. japp (churchill livingstone, 2007) ww

143

Upload: others

Post on 11-Sep-2021

3 views

Category:

Documents


0 download

TRANSCRIPT

Page 1: Arterial Blood Gases Made Easy - I. Hennessey, A. Japp (Churchill Livingstone, 2007) WW
Page 2: Arterial Blood Gases Made Easy - I. Hennessey, A. Japp (Churchill Livingstone, 2007) WW

lain A M Hennessey MBCha (Ho,n)'" IH='I M'eS

Senior House Officer, Ne()nol<lllnlens;lIe CoreRoyol United HosPIIaI, Bath, UK

Alan G Japp MIlCh. iHoNl SS< IHomI MlCPClinical Research Fellow in CordiologyUniversity 01 Edinburgh, UK

bLSEVlFR

Page 3: Arterial Blood Gases Made Easy - I. Hennessey, A. Japp (Churchill Livingstone, 2007) WW

CHURCHILLLNTNGSTONE

CLSEVlER

Cl2OO'l, Dsorvitr l.I.n,tro. AU rip'"~

No part of th'''I'"bhcation may u.. ..."rodll<.Wt. !;I,,,....j in ~ rWicval ~)'I~, 0<tnnmUnl~11tlany form or try .ny meam, el".:tn...ue, mf!<:hanica1.I'OOtooop)'i1le;..t«Oidi"g or oct..... ..-be....,ltnut tht prior ' .....1IlUIIioon ci ttr PubbIhenPen",,,,,,,,,,,, may be .>Ught directly hu'" FJSorrier"s Hrolth Scie>c6 Right>.U'1,aliment, 1600 luhn F. J(ennedy RlIUI""'ard. Suit(' IHOO,Philadelphia, FA191W-2IW9. USft.: phc:lne: (+1) 21 <; 2.19 3804; law' ( • I) 215 2:)9 3lI05; or, ...m.1.il;IIIlIII1tJtpa.- e k_~ ,,'" IN'\' abo C'<lIJ\f'to.w yOW"~ n.>-broe ''101!he~homrpIo~ {h"P'1 ! _ ...·,~-.'""l,b,- oIekctuq; 'Support andCQnt<K1" lind lhotn 'Copy"g1'1 and rermiso!.. ~"

fSDN·13, 978-().-41::1-1{).l14-S (M,,,,, Echboo)1SllN-13, 9'lS-Il-«3-1041311 (lnn-'.....l>l)na/ E.dih'"')

Brili$h 1.......uyCatAl..S'''''1l. in .....bllc"ho.. OauAcnto'lngue r«ord fc.w tillS book is ....a,Loble from the KTlt~<Jl Libruy

l.ibrwy 01 Consn- e.taJosb's in f>ublicatiM D.....AQtalos n-o:<oN for thd booll '" unbbie from ItoP t...ibrary of c.-W"M

"""'"N("Ott- the publisher ..... 1ho! ltuthon. _l.U'ne' any~....bility 1m any Ie,.. orinJurY ./CK~ 10~ or i"'OjA' Ii ari$mS' "'I 01 or n'Latl:'d 10 M<'use of the mal<T>~1coot.>ined If1 thos book. It is thr.o n:sp<JnlIibility of t1.. laallngprKti~"'lyin,l; on indcpl~ldenl~''''Kl knowl~ge of U'" pat~t. toOtlaDWv-lhe mllreatrn<..,1 and method of applKatlOll foe tht",,"IHnL

7\rPtoW......

~L:lE"IFR

WO,.king together to growlibraries in developing countries

--.doc¥icr.cocn I WW'W boob.,J.~ I __OAbK org

your soun:. fw boolu.jo.-na& and ndtio,.....In tt.. tMotth sclenc:es

_etse\fierheoltb.com

..ll'l'? ..P*Y_"_

,.,., •• r I

..... II "II:

I

Page 4: Arterial Blood Gases Made Easy - I. Hennessey, A. Japp (Churchill Livingstone, 2007) WW

Preface

If you've taken the time to open Arfnwl Blood Gases Mnd, l-:tJsy, youmust believe that artenal blood gases (AHtJi) are unportant. but notPntirely:,lraightforward.

We ft>rtamly agn..~ on the first pomt: ABC analysis now play:. il/1

indispensablf' role in the assessment and manilKcmcnt of pahf'nh;

wltll a huge range of acute moollal and surgu"al problems. AccuratE'ABG interpretation is undoubtedly a fundamental skill U1 modemclink'" medianl::.

On the SI.'('ond pomt, we hope this book ran bto of a~sislance.

Throughout. our aim~ have ~,\ to emphasi<:l' the key concept~,

fnrus on priKtical and useful aspects of ABG analysi!! and dvoidextrant"Ous detal1. \Vc believe lThlny medical and nursing students,JWlior doctors and specmhst nurses will benefit from a dear, ronciseguide to performing thl? te<:hnl4uc and interpN>tmg the results

lain A M HeJUlCSSCYAlan Glapp

Page 5: Arterial Blood Gases Made Easy - I. Hennessey, A. Japp (Churchill Livingstone, 2007) WW

Contents

Prefcx:e v

Acknowledgements V"

Port 1 The ABG explained 1

1.1 Introduction 3

1.2 Pulmonary gas excnange: the basics 41.3 Disorders of gas exchange 181.4 Acid-bose balance: the basics 26•

1.5 Disorders of ocid--bo$e balance 361 6 ABG $Ompliog technique 42

1.7 When and why IS on ABG required? 481.8 Common values 501 9 Making ABG inlefprctolion easy 52

Appendix 56

Pori 2 The ABG in practice 57

Cases 1-25 58Answers 109

l<>de. 135

Page 6: Arterial Blood Gases Made Easy - I. Hennessey, A. Japp (Churchill Livingstone, 2007) WW
Page 7: Arterial Blood Gases Made Easy - I. Hennessey, A. Japp (Churchill Livingstone, 2007) WW

1. 1 ..

INTRODUCTIONArterial blOOl"l ~,\S (ABC) analysis ~fers 10 the mcasuremE'nt of pi Iand the partial pn:s~.m:s of oxygen (Oz) and carbon dioxide (Co,) inartt>rial blood. From these values we can ass....s ttwo state of tJdJ--busebolanCt in blood and how well lungs are ~riorming their job of gll5,:xc}Ul11gt.

Already the~ afC questions: what is meant by 'acid-base status'?What is.:l 'partial pressure'? Why do they matter? It helps to breakthings cJown.

Part 1 of this book is designed to answer the:;e questions. We startwith a few p<lg~ covering the b.1sic CSSli:Jlhals of respiratory and aadbase physIology: plel1St' do not skip them! If you understand these COT@

concepts, thf' rest wlll follow seamles.s1y. Part 1 abo t:xplainll how,when and why 10 obtain an ABC sample, bcforeconduding with a'iimple step by-step guide to intcrpretin~ABG data.

Part 2 tht-n allows you to put aU of this into practice with a senesof case ~arios involving ABG analysis. You may already have amethod fOT interpreting ABGs but we lUge you to try our system (setNIt in section 1.9) that offers a logical, rndhodical and consistent wayof approachmg ABGs.. By sceinM: how this system can idt'Jltify all of~ majOr patterns of ABG abnonnalities. we hope you will gain the

necf"<i..o;ary confidence to apply it in clinical practice.

3

Page 8: Arterial Blood Gases Made Easy - I. Hennessey, A. Japp (Churchill Livingstone, 2007) WW

PULMONARY GASEXCHANGE: THE BASICSOUf ~lls use oxygpn (0,) to generate CJ\cq:;y and produce carbondioxide (COz) as waste. Blood <;upphes cdb with the O2thfoy needand dears the unwanted COt. This p~<;d~dson the ability of

our lungs to f'nnch blood with O:z and rid it of Co,

Pulmonary Ras rxcllatlg' ",fel'S to the ttdusfer of O:z from theatmosphe~t~) the bloodstream (oxygenation) and CO2 born thp

bloodstrp-am to the aunosphcl'c (C02 e-limination).

TIle cxchan~e lakl"S plal'"P between uny air sacs call1''d alvroll andblood. \'t$scls called mpilfl1n~.Because~y carll haxe exlTf'melythin walls and rome into very d06E' conmct (the al\'coIar-capillarymembrane), Co, and O:z arc able to move (diffu~)betw~ them(Figure 1).

AF~ure 1 Ra.pimloty (Jnolomy.

-'"bIollctliole

I

02 and co, lranSlBroccurs at the aM!dar

B capIafy l,lfl!OOrar,I

Page 9: Arterial Blood Gases Made Easy - I. Hennessey, A. Japp (Churchill Livingstone, 2007) WW

PULMONARY GAS EXCHANGE: THE BASICS

ABC.s help us to assess the ~ltectivencssof gas exchange byproviding measurernt"tlts of the partial pmiSurn of O2 and COz inarterial blood - thE" PflOz. and Pa~.

Partial pressure dcscribe5 thE" contribution of on~ individual gaswithin a gas IJ\i.Jl.ture (such as air) to the total pre;sure. When a gasdissol\l~m liquid (e.g. blood), the amount dissolved depcnili; on thepartial pt'PllSUTe.

Nole

~. partial pressure of 0 1

PalJl '" partial pre$5ure of O 2 in D.rl~ri.l blood

Gases move from areas of higher partial pressure to lowE"I' partialpICSSW\!. At the alveolar-<apillary membrane, air in alveoli has ahigher 1\7 and lower Pro, than capillary blood. Thus, Oz molecuJe:o;moVE" from alveoli lu blood and CO:z molecules move from blood toalveoli until the partial pressures arc equal.

A~on···... P_uU""

AI :leO 1eYeI, otmospherk. pro!oSUfe (klkJl p"euure of gases in thealrno$pn..1 - 10 1 lPo Of 760 mmHg

o? oompriSC5 21 %of air, so the porliol pre$sure of~ In oir• 21% of otmospheric. pnmure.21 kPoor l60mmHg

Co, mcJI,;~ up fV1l a til"/)' frodioil 01 air, so the pamol pressure ofC~ in

in~,ed aIr IS negligible

Page 10: Arterial Blood Gases Made Easy - I. Hennessey, A. Japp (Churchill Livingstone, 2007) WW

f'Ut.MONARY GAS eXCHANGE; THE 8ASICS

CARlON DIOXIDE WM''''ATION

Diffusion of co, from lhe bluodstrPam to alveoli is so dficient that

COl elimination is actually limited by how quickly we can "blow­off" the CO210 our alveoli. Thus, the PilC02(which reflE'Cts theoverall amount ofC~ in arterial blood) is dttermined by al'Ot.'OlarDrnhlahon - the total volume of air transported bctwt.:cn alveoli andthe outside world every minute.

Vmtilation IA regulated by an area m the bralnstf>m called therespiratory centre. 'This area containS specialised rca::ptun. that sensethe PtlCOz and cvnnect with the muscles involved in breathing. Ifit is abnormal, the respiratory centre adju~ts the rate and depth ofbreathing acrordingIy (Figure 2).

Normally, hmgs can maintAIn a normalP~ even in rondltioru;where Co, production i!'l unusually high (e.~. sepsIS). Consequentlyan increased PIKOz (hypercapnia) always lD1plies reduced alveolarventilation.

Key point

PaCo1 is controlled by vtnlil.llion ~nd the level of v..ntilation isildju5ted to m.aintilin PaCo1 within tight Iimils..

•I

Page 11: Arterial Blood Gases Made Easy - I. Hennessey, A. Japp (Churchill Livingstone, 2007) WW

PUlMONARY GAS EXCHANGE: THE BASICS

~Paco2" +V&nlilatioo

t PaC02" tVentla\loll

Figure 2 Control of ventilation.

~ note on ••• hypoxic drive

In patients with chronicolfy high PoC02 levels (chronic hypercapnia), the

~iali-ed reo:;eptors thot deled CO2 kmlls can become de$ensilbed. The

body then relies on receptors that detect the POiJ-l to gouge the adequocy ofventilation and low POOJ becomes the principal Vflnlilotory stimulus. This isreferred to as hypoxic drive.

In patients who rely on h~ic drive, overzealous cOfrection ofhypoxoemio, with supplemental O:z, may depress ventilation, leading

to a colastrophlc riM in POCO:!. Patients with chronic hypercapnia

must therefore ~ given wpplementol O 2 in a controlled fashion with

careful ABG monitoring. The same does not oppfy to potienb with ocut.hypercapnia.

7

Page 12: Arterial Blood Gases Made Easy - I. Hennessey, A. Japp (Churchill Livingstone, 2007) WW

PULMONARY GAS EXCHANGE: THE BASICS

HAEMOG~O.'N OXYGEN SAtURATION (SO,)

Oxygenation is marc complicated thanC~ t'liminabon. 1lte firstthing to realiSE' lS that the Po:!. does not actually teU llS how much~ is in blood. It only fTlf'asures free, unbound~ moIecules- a tinyproportion of !he total

In fact, almost all 0,. molecules in blood are bound to a protem calledhamwgfobin Hb (Pigure 3). Becau~ of !.his, the amount of O2 In blooddepends on two facton.:

1. Hb concentration: this determines how much O2 blood. has thecapaCIty to carry.

2. Saturation of lib with 02 ($0,): this is the pcrccntaj.;c ofavailable bindin~ sites on lib that ront3m an 02 molecule - i.e.how much ofthe carrying capacity is btrng used (Figure 4).

Note

Soz:: O 2 wtueation in (;my) blood$Q:O:!. "" O2 "tueation in Q:rlmal blood

It. note on ... pulM~i t

So::>:2 con be me<»ured using 0 Pfobe (pul!oe o..imelerl opplied /Q thefinger or earlobe. In most case$ it Pfovide$ odeqUQte informatioo 10 901.19­Q,ll,ygenarion, but it iller.s occurate witn saturatlon1 below 75% and

unre!iQble when plIfipherat perf",lion il poot. Oximetry 00e$ not provide

infoflTlOlion on f'acQ) w ~hwklllQr be u~ 0$ 0 wbditute for A8G onolyJisIn ventitololy impoir",..l.

Key point

.Po,. is not a m('uu~ of the unount of O2 in bJuod - ultimately theSaO:!;;md Hb concentration determine the D,. confent of arterialblood.

Page 13: Arterial Blood Gases Made Easy - I. Hennessey, A. Japp (Churchill Livingstone, 2007) WW

PUlMONARY GAS EXCHANGE: THE BASICS

O:!tnnlklHb(M)

Ft,ure 3 Relative fXoporliom of Ire. 0, moIecot.s and O 2 moIecuUubo<md b hoomoglobin in blood.

HaemogIobil Fre9 QIl)9lll Boln:I axygen1TlOIea.M rnoIecUe ~ •

,• !,i • ,,!

•,, • I,

" • ,!,I

I ••

I,,

• I•

11::'=1 11~~=1 I~=IFigu... 4 Haemoglobm OX)IQ~ sol!Jfation.

Page 14: Arterial Blood Gases Made Easy - I. Hennessey, A. Japp (Churchill Livingstone, 2007) WW

PUlMONARY GAS EXCHANGE: THE BASICS

OXYHAEMOGLOBIN DISSOCIATION CURVI

Vk now know that the amount of~ in blood depenWi on the HbCOTKt.>nlcation and the~. So what is the significance of the PQ:l?

Po;. can be thought of IlS the driving force for O! molC'Cull.'$ to bindto Hb: as such it regulates the Soz. The oxyhapmoglobin dissociationCUC'Ye (Figure 5) shows the So,. thai will result from any giV01 Po,..

In genern,~ higher the Paz, the highel" the Sov but Iht CUrtlt is notlittttlr. Tfw. green line is known as the 'flat part of the cwve': changesin~ over this rangf' have relatively little effect on the 50:1. Incontrast, the red line is known as the 'steep part of thf' curve': evensmall c.hanges in PO]. over this rangf" may have a major impact on~.

Note that, with a 'normal' PIlO:l of around 13 kPa (100 mmHg), Hb is,more oc less, nUlximally stlhmdm(~> 95%). This means blOl.X1 hasuwd up its D,-t:acryinK capacity and any further rise in Pao, willlWtsignificantly increase arterial D.z content

Key point

Po" is not the amount of O 2 in blood but is the driving force for5.lItunting Hb with 02"

I.

Page 15: Arterial Blood Gases Made Easy - I. Hennessey, A. Japp (Churchill Livingstone, 2007) WW

PUlMONARY GAS EXCHANGE: THE BASICS

100Cww shifts to IIfl wtleft:!pH.IPro,*lemperaIln,

80 RJDPG

Curve sIllfta k) righI wMri:IpH.lPoCO"i'twnper.lIllI'e,12,3OPG

"o+-,-~-=~_~_~--,_,~

o 5 10 15 20 80 ~~,o 37.5 7S 112.5 150 600 ~rrmtg

Figuro S ~Jobjn disso:iotioIl anotl. The CUfVe defines the,J.m.,,,sJ.ip beMun Po, and the percfflJloGe so1uI"ation 01 haemoglobmWIth 0")'9$l (~_ NoIe the sigmoId sho~: it is relolive/y Rot when P01 's >80 mmHg flO 6lPa) but sleep whoM Po, foils below 60 mmHg fS lPo)

Key point

When Hb approaches maximal O2 saturalion, furth~r increalles inPaz uu not significantly increase blood O! cuntent.

"

Page 16: Arterial Blood Gases Made Easy - I. Hennessey, A. Japp (Churchill Livingstone, 2007) WW

PUlMONARY GAS EXCHANGE: THE BASICS

AlVEOUR Vllffll.ATlON AND~

We hav*, now seen. how P«>z re,;;uJa~ the ~. But what dcl'erlniJ'w><l

P«>z'

There are three major factors that dictate the Pac,;:

1, Alveolar ventilation

2. Matching of ventilation with ~rfusion (VIQ)

3. ConCUltntion ot 0 1 in inspired air (FiOa)

Alveolar ••ntilation

O:z moves rapidly from alveoli to till: bloodstream - so lhe hIgher thr

uluwwr Po" Ihe hixhtr the PIlOz·

Unlikp air in the atmosphere, alveolar air rontain.'l significantamounts of CO:z (Figure 6). More CO:! means a lower p~ (rememberthe partial pressure of a ga" reflects Its share of the total \'olume).

An mcrease in alvoo1acventilation allows more CO:z to be 'blown oH',resulting in a~ alVl."'Olar ~. If, on the other hand, vcntilatiorl.declines.L~ accumulate; at the cxpenw of O:z and alveolar~falls.

Whereas hyperventiation can mcrea!le alveolar pO:! only slightly(bringing it closer to the Po:z of inspin>d air), there is no limit to howtar alveolar f'oz (and henceP~ can fall with inadequate v.:ntilation.

12

Page 17: Arterial Blood Gases Made Easy - I. Hennessey, A. Japp (Churchill Livingstone, 2007) WW

PULMONARY GAS EXCHANGE: THE BASteS

75"""

.....75"NftgIl.

"'-'""""".....- .......,tgur. 6 Composrlfon 01 inhaled ond ..."haled 905& at voriou$~$ oJrespiration.

K.y point

Both uxygen.ation .and COl elimination depend on alveol.uventil.ation: impail'e'd vcntiJiiltion cau5e5 PliO, to fall and Plica, turi,..

13

Page 18: Arterial Blood Gases Made Easy - I. Hennessey, A. Japp (Churchill Livingstone, 2007) WW

PUlMONARY GAS EXCHANGE: THE BASK:S

Ventilation/perfusion mismott:h and .fhunring

Not all blood flowing thmugh the lung meets wt>U-venhlat~alveoli:md not all vt:ntilatcd alveoli ilrt' pt'rfused with blood - E"Speeially Ul

tht> prest>ncp of lung disc~.This problem is known as ventilation Iperfusion (V /Q) mi'lmatch.

lmagme if alveoli in one area of lung are puorly venbJittPd (e.g. ducto collapse or consolidation). Blood pitssmg thot' alveoli n>tum.~ tothe arterial circulation With Ie::;:; 0, and more ('01 than normal Thisis knuwn as shuntingl

Now, hy hyperventilnting. we can shift more air in and out of ourremaining 'good alveoli'. nUs allows them to blow-uff eXlraC~ SOthat the blood passing ttwm can offload more CU:!. 'The luwer Co,in non-shuntrtl blood compertSittH fOf" the higher COl in ..huntedblood, m;untaming tht! poleD,.

The :,ame docs NOT apply to uxygt!J\i1lion. Blood passing 'goodalveoli' is not able to carry more 0, Mause ils hM>moglobin i::.alrt>itdy maximally :xIturatcd \,nth <-1:1 (remember: flat part of curvt:,page 11). Thf' non·shunt~blood lhenofoN' carumt rompensah> for the

low O:z levels in shuntt"d blood. and thep~ falle;.

Key point

V/Q mismatch allows poorly ol()'genated blood tu re-cnter thearteriill circulation, thull low~ring Pao" and Sao2"

Provid~ ov~rall il)veolar ventilation is main~ed,V/Q mismatdtdoes not I~ad to an inC~ilse in PacOr

'Tl\(> tPTITI also applies to bluud lh"t bypasses alveoli altogelher (anatomICal~'W1IUlg)

14

Page 19: Arterial Blood Gases Made Easy - I. Hennessey, A. Japp (Churchill Livingstone, 2007) WW

PULMONARY GAS EXCHANGE: THE BASIC",S,--_

.J=~=I'''''''''O2

so,

'"so,

'''"'so,11m

--.....--

so,",,.

..­CO2

.......CO2

15

Page 20: Arterial Blood Gases Made Easy - I. Hennessey, A. Japp (Churchill Livingstone, 2007) WW

PUlMONARY GAS ~CHANGE. mr BASICS

,~ and oxygenation

The fraction of inspired oxygen (FiO:J refers to the percentage of0: m the air we breathe In. The FiOz in room air IS 21 %, bul can "*increast"d wIth supplementaJ~.

Aluw PacJ.z may result from t'ithcr VIQ mismatl..:h or inadequatevrnblabon and, in both ca~, increusi.njl, the F~ will improve thePaOz. 'l~ ~)(act r~ rcquirem..nt vane!:> depending nn how severelyoxygenation is impalrM and will help tod~ the chokP ofdeli...,crydevice (figure 8). When thf' cause is inadequate ventilation it mustbe remE>mbered that incrc3Sing riO, wIll not rev~ the rise in P~.

Supplemental~ makes ABC analysis more complex as it can bedifficult to Judgc whether the P/Kh is appropriately high for th~

F~ and, hE"llet", whl.>thcr oxygt"nabon lS impaired. A useful rule otthumb is that thp diffeft'J)(.'C betwffn r~ and POOl (in kPa) shouldnot normally be ~r~ater than 10. Howevpr there is often a d€'greeof uncertamty as to !he pl"€'Cise FiO,. and, if subtle impaU"lucol issuspected, thf' ABC should be ~..atedon room air.

OXYgMI et.livery devic.,Nasol fNO"gt. f~ < ..~. ComfOl'tol:h ond comenient FiO, n0n­

specific: depends on flaw rem t1-6 Vmin) and venhlabon

Stondard fm;" mask: fiO, 30-50% at flow rotel6-10 Vmln bulImpuKise Mot COllie CO:! relention ot flow, < 5 Vmin ('rebreolhing'). 100

not 1IJefu1 Jar providing 10- ft0:2fixH p«fOiilNilKe (high&wl~rno~ F~ 24--00%. Ddivers fUced,prediclabl. Fi02• Ideol for providing controlled, occurote O:z therapy allowCorN;ef\lfQl;QI1S

foe. meuk wiIh re••nooi. ft0:2 6IJ...8O".li.. Con ochie¥e _ higher

fiO:z with IighHifhng rnosJc Useful for ano.llefm UM in respiratory

emergencIes

fndotracheGl intubation: fi02 21-100%. Uled In soyerely un......11potierlb with very high~ requiremenls, especioMy wifh venhlolory !oillire.Patient is sedoltid ond InlI<::hookolly ventiloted.

,.

Page 21: Arterial Blood Gases Made Easy - I. Hennessey, A. Japp (Churchill Livingstone, 2007) WW

PULMONAAY GAS EXCHANGE: THE BA5K.:S

Fbled pelb Ifs ... milt:

VaoobIc perforrnarx:e mask

Figun a Oxygen deJ,WKY clevius,

17

Page 22: Arterial Blood Gases Made Easy - I. Hennessey, A. Japp (Churchill Livingstone, 2007) WW

DISORDERS OF GASEXCHANGEHYPOXIA, HYPOXAEMIA AND IMPAIREDOXYGENATJON

The above lerms aN': often used interchangeably but mean differentthmgs.

Hypoxia refer.; tu any stld~;11 whIch tissUt'S M:'riot lI1I iruzdeqlUlk supplyof 0 1 to support nOrmQIlJt7'Obic mrl/lbolisml (Figwe 9). It may resultfrom either hypoxl!emia (!lee below) or impaired blood supply totis~ues (isl.:hacmia). It is ('Iften associated with lactic acidosis as cellsresort to anaerobic metabolism.

Hypoxa~mla refers to tmy slQf~ in wIridr 1M O2rontent ofQrlmal bloodis reduced. II may remit hom impaired oxygenation (see below), lowhaemoglobin (anaemia) Of reducw affutity of ha~mog1obin for Oz(e_g. carbon monoxide).

Imp.tired oxygr:nation refers to hyporMmJII fC>.41tingJrom rtduaJ.tnlmfer of D1from lungs to the bloodstrtflm. It is identified by a low P/Kl:!,« 10.7 kPa; < 80 rnmHg).

It is important 10 note Ihc distinction betWt:'ell impaired oxygenation(which results in hypoxaemia) and IrwdLqllllteoxygcnation (whkhre;ults in hypoxia). Constder a palimt with a PlIU:!. of85 kPa. He M..'limpaIred oxygenation. suggesting the prcsmce of irnportanllungdiSE'OlSf'. NeveI"lhtllt.-ss, his Pao.z would usually result in an~ > 90%and, protlidt'd th£ haemoglobin and (llrdiac output are normQl, adequate0:l: delivery to tiS$Uell.

lit" often m.lvisabl(' to u.'\t! the term 'm.."uc hypoxia' to avoid any l't>nfusion.

11

Page 23: Arterial Blood Gases Made Easy - I. Hennessey, A. Japp (Churchill Livingstone, 2007) WW

rnsoRDERS OF GAS EXCHANGt:

5pee*-­(e.g. "'JOC*cIIl

ila....... ,).<.,. ,

iJ

t_I__~__-tHoomogIobil low /1QernagIObi'I low~

I.WlIlbIe 10 carry~ (~) (1IIp"oed(e.g. CO poisoIwlg) arygemIion)

figuN 9 Cau.seJ 01 hypo;ua

••

Page 24: Arterial Blood Gases Made Easy - I. Hennessey, A. Japp (Churchill Livingstone, 2007) WW

DI~DERS OF GAS EXCHANGE

TYPE 1 RESPIRATORY IMPAIRMENT

Typt! 1 respiratory impairment' is defined as low P~with norum!or low P~.This implies defective oxygelation despite adequateventilation. V,Q mismatch is usually responsibl4" and may tc5ultfrom ;1 numbt"r of causes (Box 1.3.1). The PIIJCO.z is often low due tocompt!J1satory hyperventilation.

If the ABG is drawn from a patient on supplM'lf>tltal Oz. the Pl7O-z maynot be below the normal rangt", but will be inappropnately low forthe FiO,.

The Sf>verity of type 1 respiratory impainnent i~ judgt..'l:1 accordingto the scale of~ resulting hypoxaemiol and, ultimately, thepresence of hypoxia (fable 13.1). Hpre it IS important to rernmilicrthe 0,. dissociation curve. Reductions in PtIOJ as far as 8 kJ'a havea relatively minor effect on Sao:z and are well tolerat~. Beyondthis threshold, we reach th£' 'steep part' of lh£' curv£' and furtherreductions in PtIOJ will lead 10 much greater falls in~ significantlylowt'ring the~ content of arterial blood.

Initial treabm......ll of type' 1 resplJatvry impairment isa~ atachieVing an adequate Pac;. and Sat~ Witll supplemf'ntal O2whileatlcmpting to COIn:d Ute W\derlying cause. In many ca5f'1l pubeOlomt'try can be used as an alternativE' 10 repeated Aac; sarnplmg tomonItor progress.

'We USl' thf. lftm 'inlpairu-."ll' ralMr than 'fai.lure' hL-rc as t~ diaSJ'OSlS 01re5pU"ilkw)' failUT# requires I Pao:z < 8 kPa « 60 rnmIiX).

Page 25: Arterial Blood Gases Made Easy - I. Hennessey, A. Japp (Churchill Livingstone, 2007) WW

DISORDERS Of GAS EXCHANGE

So;.: 1.3.1 Common CQU5e'5 of~ 1 respUoIwy lmpai.....ent· ~

PneumonioPulmonary emboli$ll\

........ho<=Pulmonory oedema

Acula osllullo

Acvte re$piratory disben syndrome

Flbrosing ohieolitis

Chronic OO'l'U<:llve pulmonary diseose

Table 1.3.1 Auesdng MYerity of type 1 respiratoryimpai,""*"t

Mild ~Clte Sovwe

POOz {kPol 8-10.6 53-79 <53POO2lmmHgj 6f>-79 40-59 <40

Sao, 1%1 ~94 7>-89 <75

at". _ken of .....e... intpo........t .

• H'9h f~ reqoJiremenl$ to maintain odeqvole Paa,

• loctic: ocidl)$i$ lindicOling tissue hypolliol

• Orgot'! dysfunction (drOWSlnen. confulion, reool fo,luf., hoemodynamlCcoIIopMl. como]

21

Page 26: Arterial Blood Gases Made Easy - I. Hennessey, A. Japp (Churchill Livingstone, 2007) WW

DISORDERS Of GAS EXCHANGE

TYPE 2 RUPIRATORYIMPAIRMENT

TYJX' 2 respiroltOry impairment is defined by a high PoX..U:l:(hypercapma) and is due to madequate alveolar ventilation. Sinceoxygenation alw depends on wntilation, the Paut is usually low, butmay be oorffial if th.. pabenl is on 'iUpplemcnlal 07' It is important10 notp that any cause of typt! I impairment lllay lpad to type 2impairment if exhaustion s\lpervencs.

Arnte rises in Parll:! lead to dangpmus accumulation of add in theblood (see ~'Clion] .4) and must be rf'\·er~:tJ. Chronic hypercapma l:>

olccompamed by a rise in bicarbonate (HCO~),which pr~rvcs aCid­base balancp. Howcwr, p"tiPTlts with chJ'()r'lic type 2 irnpaUID\.'n1who expcrienn> a further sharp decline in ventilation will also holYCa rapid rise in PaC02 tacute Oil chronic), le~ding to aCid .I.ccum\llation.md low blood pH (Table 1.3.2).

Supplemental ()2 improves hypoxaemiJ; bUI nol hypercapma, ~trf'atment of type 2 ft>Spuulory impainncnl should also includemeasures to improw \'ffitilation (e_g. w.ersal of sedation, reJjpfof airway" ~Iruclion,a';si"tetJ ventilation). Thc OVPrzealouslldTlunistration of supplemental 0,. to som!' patients with chnmic ty'pf'2 impainnent may furthcrdep~'iventilation by abolishing hypoxicdrin> (p. 7).

Pulse oximetry provides no mfurmation on PIICOz so is nut a suitahlesub:.tilutC' for ABC monitoring in tyJXl: 2 ~uatory impairment.

22

Page 27: Arterial Blood Gases Made Easy - I. Hennessey, A. Japp (Churchill Livingstone, 2007) WW

OIS<:)iOERS Of GAS EXCHANGE

Tobie 1.3.2 The ABG in dfff--.t pullern, of type 2impairment

-----Po«>, HCO, pH

A<u~ f • j

Chronic f f -Acvte on chronic f f I

Bolt.1.3.2 .~I1'l~ (QUMS of~ 2 reJPiratory.lmpoirment

Chronic O&tTur;tive pufmonory di*Jse"hho",honflail cn.d injuryKypftoscoliosis

Opiotel'benzodiozepine Iollicily

Inhaled foreign bodyNeuromusctMar disorders0bWuctn. sJeep OPI(NQ

-BoX 1.3.3 Clinical~ of hypercapnia

Drowsiness

Bounding pulseHeodoche

.3

Page 28: Arterial Blood Gases Made Easy - I. Hennessey, A. Japp (Churchill Livingstone, 2007) WW

DISORDERS OF GAS EXCHANGe

HYPERVENTILAnON

Hypervl'!nlliation leads 10 a low Pan>..> (hypocapnia) and acorre:por.ding ri~ in bluud pH (set> Sfftion lA). In chrot"Jc casesit is accompanied by a ri'it" in HC0.J, which corrects blood pH. AnincreaSf" in the ratl: olnd depth of breathi.ng is usually apparent. Alarge drop;n Pan" may lC<ld to tingling arowld the mouth dodextrt'llutics, light·hPadedne.s ilIld even 'lyncopc.

Psychogeruc hy~rvcntilationoften presents in a dram.tlic fashion,with patients complaining of severe breatWcssness and an inabilitytu tuke in enough air. It may be difficult to distinguish fromre>piratory disc<lse. Tht> ABC J>hows a low PIILUz with a normal p~

H)'peJVenhlahon abo occurs a5 a compensatory response tom~tabolic acidosis (secondMy hYP*'rvpnhlation), as dE"lCribed insection 1.4. Other caUS('5 are :ihown in Table 1.3.3.

._---Po,n or diYre55f.."

Cenlro] I'ler¥OU3 s~1em diK>rc!e'l

Anxiety u»ydq,enkl...".,..,...~

Solicylote lollicity

HepatIC cirrhosisMetabolIC(X~

101 "'" oohologyt

Pllmory

Table 1.3.3 Common causes of hyper~llilmion

I

-

Page 29: Arterial Blood Gases Made Easy - I. Hennessey, A. Japp (Churchill Livingstone, 2007) WW

DISORDeRS Of GAS EXCHANGE

SUMMARY Of GAS EXCHANGE ABNORMALITIES

T'ht> four lllai.n putt~rns of ABC abnormality in disorders of gasexchange are summarised in Tuble 13.4

Tabte 1.3.4

Poo, 1'0<0, HCO,

It' .•espualOl"f '"'PO*" ..nt

TYP" 1 , 1/- -Aculo type 2 lI- I -Chronk type 2 * II- I IHypervenlilolioo - 1 ./1

°Acvle on dlfonk; di.lingui~ From dwonic bv pre..,roc. 011 H'

A note on ••• the A-a gradient

The A-o gradient i$1hc diK.rencll bet...ee" the alveolar fIt? love<agedocrou 01~l ond fie~ in or1efiol b'ood. It tells I,IS wheIher the Pao,

is oppropriole for the level of alvcolar veolilation end is Ihen~for. a meo~ure

01 the degree of VjQ mismolch.

In proclice its moin \I$OS lie in detecting subtle IncreaSti in V/Q mlsmoteh

wiler. h ~ I~ WIll willlin the normal ronge !e.g. puknonory emboli,",'ond ideotilying the presence of V/Q milmalch in patients with Iypfl

2 rc$puafory impairment {thi~ distlnguiws pure Iype 2 respiratory

Impairment from mi:wd type I ond type 2 Impairmentl

Cob.okJtioro or the A-a gradient i~ not required for the _1'0001 in Port 2 oflIlll book but, for !hole inlafel<led, a guide can be lovnd in Appendix 1

Page 30: Arterial Blood Gases Made Easy - I. Hennessey, A. Japp (Churchill Livingstone, 2007) WW

ACID-BASE BALANCE:THE BASICSThe terms aridity and alkalinity simply refer to the «()JlreIl.trationof free hydfoR,en ions (H') in II solution. H' concentration can be~xp~.Jwro:l1y in nanomoles per litre (nmol/L) or as pi I (s­

over)..

Solutions with hIgh W (low pH) are acidic and those with lowH' (h.igh pH) are alkalinf'. The POint ilt which a substance changesfrom alkah to acid is lhc neutral point (pH :z 7, H' s 100 nnwl/ L).

An i'ldd is a 'iubstance lhat ,dea5f!S H' when It l~ dissolved in~lution Acids ttwrefotf" increase~ H' concentration of thesolutiun (lower the pH). A base i.~ a sub:.tancc lh.'\1 a«tpt.'1 H· whendis.c;olved III solution. R"lSeS therefore lower the H' concentrationof a solution (raUlt! the pH). A buIll?( is a substance that can either;lea-pI or releilSt> H' dependillJ!!; on the surrounding H' concentration.

Buffers lhcrdon> rl'Sist big changt:lll in H' concentration.

Human blood normally h.\5 a pll of 7.35-7.45(II' _ 3..~.5 nmol/L) so is ::o1ightly alkalinE'. If blood pH is below thenormal ranFje « 7.3..'), there IS an addacmia. If it is above the normal

range (> 7.45), there is an alkalaemia.

An acidosll'l .." any pnx:ess thallow~blood pH whereas an alkalosIS

is any pt'ClCeS6 that r3lSeS blood pH.

2'

Page 31: Arterial Blood Gases Made Easy - I. Hennessey, A. Japp (Churchill Livingstone, 2007) WW

AClD-aASE BAlANCE: THE 8ASICS

What as pH?

The pH (power 01 hydroget'l) Kale l~ a ~Implln.d 'W'a'f of expreuing large

~ in H' =ncerflluhoo, though if you'... not corne ouou jf before you

might think. II 'NO$ d.t.iglMd iuu to confu~ you!

It i$ a MgUtive logarithmic KoIc fFigvfe 101. The 'negative' meon$!hut pH~ get lower as !he H' concentration increoMli flO a pH 01 7 1 ;$ more

OCIdic thon 7.21. The 'Iogarilhmic.' meons thol a Mlift in pH by one numbel

r.prelents a 10-f01d chonge in H' concenlfolron (so 7 Ii 10 times more

ocidic than B)

Why i. aci~s. bolance important?

J;or cellular pI"0CCSS4.'S to occur effiCIently. Ihf. II' concentration lJ\USl

be kcpl within tight limit!!. Failure to maintain pH baJarn.."C Ical.!s 10lIlefficient cellular reactions and ultimately death (Figure lO).

..pH 6.9 7JJ

"" 6J.l I~'5.8

735 1.45

7.3 I 7A I 75 1.6 71

I

Sl :5.8 1~.6 25.1 20.0

" 35

12I.'

100'16

Alu'csis... , is

..... ,. pHM """

27

Page 32: Arterial Blood Gases Made Easy - I. Hennessey, A. Japp (Churchill Livingstone, 2007) WW

ACID-BASE BAl.ANCe THE BASICS

MAINTAINING 4CID-8ASE BALANCE

What generates H+ ions in our bodies?

n.e breakdown of fats and SUgMS for energy generates~ which,when dis:dvro In blood, fOf"DlS carbonic acid (see Box on page 211).

M~abolism of pmtPin produces hydrochJonc, :lulphurit: and otherso-called 'metabolic acids'

H· ions mu.st, therefore, be rcmovL'd. to maintain normal blood pi!.

What removes H+ ion. from aur badies7

Respiratory mechanisms

Our lungs are responsibl~for removillK CO2_P'ICO:!, the parti..,1pressure of carbon dioxidt> in our blood, is determinoo by alvL'Olar

vCl1tllo:ation. IfC~ production is altt"red, we adjust our breathing10 exhale more or k'SS COg as IlI.'."CSSary. 10 maintain PlIC'O;z withinoormallimifs. The bulk of the acid prodlK.L'd by our bOO.ies is in the

form of COz, so it is our lungs that excrete the vast majority of theo:aud load.

Renal (mefobollc) mechanisms

The kidneys art' r(>$poru;ible for eXcrelmg rnmbolic acids. TIleY~'l,;rctc H' ions inio urint> and reabsorb HC0.'l from urine. HCo, io:t abase (and therduA:' <1<.:CCPlS H' ions), so it reduct'S the ("onr('ntrationof H· ions in blood. The kidn~yscan adjust urinary H' and IlCo,excretion in responSE' to changes in metabolic led pnxluL1ion.

Page 33: Arterial Blood Gases Made Easy - I. Hennessey, A. Japp (Churchill Livingstone, 2007) WW

ACID-BASE BAl..ANC~: THE BASICS

MAINTAINING ACID-BASE BALANCE

The l'Cnal and respiratory syst..ms operat", Jomtly to maintain blood

pI-{ within normallirruts. If one system IS ovtrWhelmcd, leading:to a ch;mge in blood pi I, the otl'llo'[ usually ddjUSl$, automatically,

to limillhe dlSturh;mce (e.g. If kidneys fail to ~cretp m~abolk

acids, ventilation is UKTlo'a..'led to exhale mure C00. This is known all

wlilf!t-1ISQUm/

Importantly, lvrnpensatOry change1l in respiration haprt"" over

minules to hours, wherc<li:> mt>taoolic rcspou!>eS tak(> days to develop.

Jwt one equation•••~---

This one eqoolion is c:rucial to understanding ocid-ba.se bolonc:.;

~o + CO2 +-+ ""Cal .... H' + He0s

Firs/ly It show51hot C~, when dissolved In blood, becon'lfts on <rid_

The mOle COz added 10 blood, the more H1CO, lcorbonic oc,dl is

produced, wIloch dissociates to release free W ions

S8ct:Jldy, it Pfedicts Ihot blood pH depends not on the obsolut.t ornoontsor Co,; 01 HCOl present but on 1M 'Otio of CO2 10 HCo.,- Thus, (] change

in C~ will not Ieod to a c:horoge in pH if III, boIonced by 0 c:honge in

HC~!hot p'esetvei Ihe ratio (and vice ....nal· SinceC~ is c:ontrolled byrespirolion ond HeO, by renol~, !his expkJlnl how cornpensotooncon pr~t c:honges in blood pH.

29

Page 34: Arterial Blood Gases Made Easy - I. Hennessey, A. Japp (Churchill Livingstone, 2007) WW

AClD--BASE BALANCE: THE BASICS

DISrURBANCES OF ACID-BASE BA~LANCE""

An acidOl'iis is any process that acts to lower blood pJ I. II it is due to;; lise in P~0z.it is calied a r£~pirator.LJ acidosis; jf it is due to any otherC<lUSC, then HC03 is redw.:oo and it is called a metabolic ucidosis.

An alkalosis is any process that acts to increase blood pH. If it is nueto a fall in PQ~, it is called a respiratory alkalosis; if it is due to anyother cause, then HC03 is raised and it is callt..-d a metabolic alkalosis.

Po::.Q:1 roiledPac~ lowHC03 raisedHCOJIow

- Relpirotory acidosis_ Respiratory oU:olo~i~

• Metobolic olkolmi~

_ Metabolic:: ac::ldalis

Add-hase disturbances can he considered as a set of scales.

Normol acid-bose balance

Wht.."11 i1cid--base billance is entirely normal, with no alkalotic oracidotic pressures, it is like haVing a set of scales with no weights onit (Figure 11).

Uncompensated acid-base disturbance

VVhen an acidosis or alkalosis deve.lops, the scales become.unbalanC'f'd, leading to acidaemia or alkalaemia respectively.in Figure 121herc is a primary respiratory acidosis with no opposingmetabolic process.

3.

Page 35: Arterial Blood Gases Made Easy - I. Hennessey, A. Japp (Churchill Livingstone, 2007) WW

ACID--aASE BALANCE: THE BASICS

7\/ \, .

I

=

"

Page 36: Arterial Blood Gases Made Easy - I. Hennessey, A. Japp (Churchill Livingstone, 2007) WW

AClD-8ASE BALANCE· THE BASICS

COMPENSATED ACID-BASE DISTURBANCE

As described earlier, a respiratory or metabolic disturbanre is oftenC9",;>rll5(llro for by i'ldjuslmMt of the utl~ system to offsf1' theprimary Jisturbiln..:c.

Figures 13 and 14 represent two St.'Cnarios in which the hU1~haveresponded to a primary metabolJc aad~is by increasing alveolarventilahon to chminalC more CO2 (compensatory respiratoryaJkillo<;i<;,). In Figure 13 an acidaemia pE'rsic;ts despite u.IInpcns;!tion(partial compmsation); In Figure 14, blood I'll has rrtumed tu thenorm.u r.lngc (full cOffipens;ttion).

Whpn facffi with ~uch lUli\BC, how can wp tell wru<:h iSlhe primarydisturbance and which is t~ rompcnsalOry pf'OCt"<lS?

The fir.,t rule 10 remember is that llwJcumprnSlltion dots not (ICCIlr.

The midpOint of the acid----basc scales lies at a pH of 7.4 (H' 40). Ifthe scales tip loward ilCIdaemla (pH < 7.4), this <;,uggests a primaryacit.lotk process, it they tip mward alkalacmia (I'll> 7.4), a primaryalkalotic pruc(,::>s is likely.

The 5('('ond rule 15 that the patirnt is /IIQre imporltlnlthnn lhe ABC.When considering an ABG, one must always lake acrount of theclinical ('\.IIltexl For exampw, if the pabent in Figure 14W~ diabetic,with hIgh levt>ls of lc.ctoncs in the unne, It would be obvious thai Ih...melabolic acid~ls was a primary pJ"O("~ (diabetic ketuadJosis).

It ;$ not~ «)$Y kl~ two pnmory oppoo$Ong p«X:flM$ froma «WTlperuoted d,w.bonce ;. more pr«:iMl method Ihon !hotde~obo.... involvM cakvlating Ihe ellpoc.oo compensatory response lorony g,ven primary di1turbonce. Howevtlr the~ colculations are usually

unneceuaryond ale not required for the case~r~ in Pan 2.

Page 37: Arterial Blood Gases Made Easy - I. Hennessey, A. Japp (Churchill Livingstone, 2007) WW

ACID-BASE BALANCE: THE BASICS

-----,

""""'""""""

-

rtgUre 14 Fully compemoNJd meIOboIic ocidc)JIJ

33

Page 38: Arterial Blood Gases Made Easy - I. Hennessey, A. Japp (Churchill Livingstone, 2007) WW

ACID-BASE BALANCE: THE BASICS

MIXED ACID-SASt DISTUIlBANCt

When d primary re.piratory disturbance and primary metabolicdi<:turbance occur simultaneously tht.-re IS :.aid to be a minoJacid--base disturbanct'_

If thoe two pnx:t.~~oppose each other. the pattern will bE'simiiartoa compensated aCld-base dlsturoanre (FigUl\: 14) and the resultin~

pll d('ran~ementwill be minimised. A good@xample c. salicylat~poisoning. where primary hypeiVentilation (respiratory alkaIMl<:)and metabolic acido:.lS (~hl:ylilte is acidic) occur independently.

On the other hand, if the two processes call~ pH to movt: In tht:sall1~ diro;:tioll (metabolic acidosis and rl:'Spiriltory addos!..:: ormetabolic alkaloMs and reo.piral.ory alk.1.1osis), .1 profound acidaentiaor alkalaemia may N"iult (Figure 15).

The nomogram

An alternative way to analyse ABG~ ~ to usc the acid basenomogram (Figu~ 16). Ky plotting the PIK.u, and H· /pH valut.'S unthcABG IlOJll~ram.most ADGs can hoe analy<;ed. If the plotted pomtlleO oubide tilt: dt.'Sigfl.ltl..'<l areas, this implies a mixed disturbance.

Page 39: Arterial Blood Gases Made Easy - I. Hennessey, A. Japp (Churchill Livingstone, 2007) WW

ACID-8ASE BALANCE: THE BASICS

l'et3tdca. ll NS

Ftgure 15 MIxed respirolory and melobolic ocKlasis.

(H'J-<'<1.0 100 5 10 15

OJ "1.1 80 30

70Nofmal1af9l12

80

1.3 SO

" 4C

" 30

" 20

10

00 2 , • • 10 """0 15 30 45 80 " OJ """""'\erial PaC~ (k,Pa)

Figu,.. 16 The ~rom_

3.

Page 40: Arterial Blood Gases Made Easy - I. Hennessey, A. Japp (Churchill Livingstone, 2007) WW

DISORDERS OFACID-BASE BALANCEMETABOLIC ACIDOSIS

A metabolic addosis is any pl'Ol.."CSS, otlu:r tlum a rise i1lp~ thai actl10 lower blood pH. It may occur through accumulation of ffiE'tabolic.:lcid~ (excess mgeshon, 1nrrf'.1"oed production or n>duced. renalexcretIOn) or through exct'SSi"'e loss of base (HC~).c.l1culaling theanion gap (sec over) may help to ~tab~h the caU1Ie uf a metabolicacidosis.

Metabolic aridOf;is is recognised on an AI}(; by low HCo,(and negative bas<' exC('5S (BE». TIlcre is normally a rom~turyincrease in aly\.'OI.tr vt!1\b1ahun to lower l'at.:0:!. If resprratorycom~bonIS overwhelmed, an acidaf'mia will result. ~.~ty

must be judged aN'ording to both thto lmderlying process and theresulhng acidat'mia An J ICO;, < 15 mmol/L (or BE < 10) indiCi1t~

a severe acidotic process whereas ... pH < 7.25 (H~:> 55) constitutesserious at.idaenua.

The dominant symptom in metabolic acidosis is oftenhyJX"f"entilation (Kussmaul's respiration) owing to the re.piratorycompcnsatiun. Otht:r ~Igns are fairly non-speafic or related to thf'wu.lerlying cau~ Pmfound acid:u>mia (pH < 7.15; I'· > 70) may leadto arculal0'1' shod:,~ dysfuncLion, and, uILimatcly death.

Specific caU$CS or mdabolic aadlY.>1S are chscussed in greater detail illthe relevant Casel in Part 2.

36

Page 41: Arterial Blood Gases Made Easy - I. Hennessey, A. Japp (Churchill Livingstone, 2007) WW

DISORDERS OF ACID-BASE BALANCE

The anion gap

In bkxxl, posilivefy charged ions (cations} musl be balanced by negotivefy

chorged ions (onions} 10 maintoin flhtctron8lJlmlily_ Bul when lhe moin

colions [Na' + K') are compared with the main onions (et" -+- HCOJt there

appears 10 be a shortoge 01 onions or an anion gop.

Anion gap =(NG + K) - (el + HC03) {Nonnal =10 - 18 mnlOl/L]

The eop b mod.. up of unmeo""red onions wei, u, p1,u>pIKJ'" UIlJ >ulpIKJIt;

and negativefy cholged proteins (these are difficult to measure}.

A raIsed onion gap (> 18 mmol/L) therflfarfl indicates the presence 01increased unmeasured anions, "9. !octole, solicylalc.

A note on .,. Iodic: ocido!5i!5

This cornman cause 01 melobolic <xidosis <Xcurs whCfl lissues roceive on

inooequa", wpply 01 0 1 due 10 e!lh"," hypo.wemia or impaired perfusion.

Norma! oelobic metabolIsm {which relies on 0 21 is then reploced byanaerobic metabolism (which generales lactic odd). lactic acidosis;s

l!Ierelore 0 markIN 01 tiSSlIe hypoxia ond a useful indicator 01 ~firy in

"voriety of conditiOM, ,,,dudir>g \oeVC<C hypoxoemlo or shock from any

couse

37

Page 42: Arterial Blood Gases Made Easy - I. Hennessey, A. Japp (Churchill Livingstone, 2007) WW

R.spOllie 01kidney

DISORDERS OF A(lD-BASE 8ALANCE

METABOLIC ALKALOSIS

A metabolic alkalosIs Co uuy process, ofh" than II fall ill p",{~. that aetsto ulcrcasc blood pH and IS chtlr<lClcrised hy a ri!X: in plasma HeO,.

I{p<;prratory compensation (f Pm.:(':!) occurs to limit the n.-sulting.llkal.wm13 but J..:. limited by the nf"f"d to noid hypo)l'a~mJa.

~ of H' ions may imtlate the process but the knlncys have- huge~ope to correct threatenl"d alkalosi::. by incI'f:'a~ingHC03 excretion.Factors that unpair this rcsporlSf> are therefore also nPCt.'Ssary. TIleusual suspects are depletion of chloridp (01, polaSSium (K·) and!WXlium (Nol') ions - in most C<bL'S due to f"ither !!Iu::.taincd vomiting(hgure 17) or ..hurctic dmgc:.

, 'f't'Iwl a- IS n :sto1~ \he klcNy ITlU$l Me9 HC~- " pmerweI«:troneIAroI (the~ balance of positive and negaINe ctages "the bocttl

"

Page 43: Arterial Blood Gases Made Easy - I. Hennessey, A. Japp (Churchill Livingstone, 2007) WW

DISORDERS OF ACIQ.....8ASE BALANCE

SUMMARY Of MEtAIOLlC ACIl>-IASE DISORDERS

10", 1.5.1 Metcabo6c oddosis Uow HCO:s) ,

With ralAH onion gop

toclic ocldosi1 (e.g. hypoxoemia, lhod, sep:5i1, infordion)Ketoacidosis ldiabetes, ~Iorvolion. okohol ex~sslRenol foihJrftPoiwning {olpinn, melh<lnol, elhylelWl glycol)

With normal an'on gop

R:croI tubular «!domDiorrhoeoAmmonium chloride i~ionAdreool inwfflclClnCy

VomitingPoIlllsium depletion (e.g. diurebc.1JCw.iog'~ ~yndrome

Conn', tyndrome !primory hyperokbteroni~J

39

Page 44: Arterial Blood Gases Made Easy - I. Hennessey, A. Japp (Churchill Livingstone, 2007) WW

DISORDERS OF ACID I\Asr BALANCE

RESPIRATORY ACIDOSIS

A r....pir.1torv actdo,sL::ll:>, ::,wlply, Uti incFt'u:'{' itl Plfl..~. Sinc(' '-- ( )1

dissolves in blood to t(\rm carbomc add, th1::. h<e; tll.,- cUcct oj

IU~,"crUl:o<,pH (fH iClM

~,)rmally, lun~ are ablc to im.:reasc \ Cliliialion to mamtaln • n,'"nnalPz:. - ('\'f>T1 in rondlhoTl.<i of mcrt:"il~~CO. prouuction (e-F;- ~i~

Thus. respiratory <1Cld(K1S ;'tlwa'\i~ lmrhp<; ad~ of t\.'i.h....C'dalvet.,Ia.r \ clltilaliOll Th' m.1\ occur from ao'! ('all,*, of type ZI'f'''piratory tmpalTlJlcnt b~ ::;c<;tu.."l 1.3\ ur to CQuntt>fitct a mf'taboh

,11kalosis

RI:SPIRATORY ALKALOSIS

A rl>SPllMl1ry ill!.;IlIMI::i III ii n ~~. ,n P anJ 15 cau~'<.i bv

aJvl'\llar h,,~rvmtilatlon 'rima.... ,U"t'li UKlude. Dalll, an.'Ilcl

h) pen ,,·lltil<1.1101l syndwme i(>Vt"f b'e;lthk>AAnp<:<; and hypo'l.M:"ru.1

It mav '1lso occur to IOUIlb:ral,; U\ ao

Ac:u~ ven.us (hroni< respiratory CKido~. _

SIflC& metoboitc compenloOtOfy ''-''''fX.''"'Cs l<J~e dnp 10 develop, ocule

re~;rotoryoc,do,i, 'olmos! ofwoys utlcompenw/<..'Ci. l.....ditl(l rnnidly

lu profo<Jnd (md don.g&rous acidaemio, lndeed, on opposing m<.-1uboli,

olkoloSIS wggesh Ihol u rl'sp;mlory q<cidosis musl hove bMn present

for some time. In other wOld~. the fl\C>(:IIC(· ,,/ mptaoolic compen$OtlM

di$ling<JIsMs chronic from o<:Ule Iy~ 2 -venh!otory IUllme j,."..

sedi"," I 3}

40

Page 45: Arterial Blood Gases Made Easy - I. Hennessey, A. Japp (Churchill Livingstone, 2007) WW

DISORDERS Of AC~ASEBAlANCE

This is the must dang~rouspattern of add-base abnormality. 11leads to profound acidac:mia as th~t'E' Me two simultaneou!I acidoticprocesses with no compensation. In clinical practice it is often dueto severe ventilatory failure', in which the rising "1lCOz (respiratoryacidosis) i:. accompanied by a luw PiIOJJ ff'Sulting in Lissue hypoxiaamI consequt'nt L1.clic acidosis.

..

Page 46: Arterial Blood Gases Made Easy - I. Hennessey, A. Japp (Churchill Livingstone, 2007) WW

ABG SAMPLINGTECHNIQUEBefore you can interpret an ADG you must, of rour::.e, obtain a:.ample of arlen",1 blood.. 'llie {ollowin,; steps should be~ as agwde but the best way to Ipam is at the bedside with expf'nencedsupervision.

BEFORE SAMPLING

• Confirm the 1It.'t..>d for the A tjG and identify any contraindkations(Box 1.6.1).

• Always record d(>tails of O:!: therapy and respiratory support(e_g. ventilator settings)

• Unless results are requi.n.'d urgently. allow alleasl20 minute'Safter any change in~ tlu:rapy before samphng (to achieve asteady state).

• Explam 10 the patient why you 3r'f" domg the test, what it involvtSand~ possible complications (bleoeding.. bnusmg, arterialthrombosis, infection and pain); then obtain consent to proceed.

• Prepare the m..~ equipment (hepari.niscd syringe withcap, 2D-22G needle, ::.harps disposal container, gauze) and donuniversal precautions.

• Identify a suitable site for sampling by pUlpilling the radial.br.lchial or femo'd.l artery (Figurf' HI). Rouhne sampling should,initially. be attempted from the radial artery of the non-dominantarm.

02

Page 47: Arterial Blood Gases Made Easy - I. Hennessey, A. Japp (Churchill Livingstone, 2007) WW

ABO SAMPUNG TECHNIQUE

A

rlgU"e 18 At~roJpvnclure sife5

8

:"C"-...........

lox 1.6.1 Conh'OindicaticMutoAaG~si,·

Inodequote collolerol ci,culollon ollhe punetut. 5iteShould not be perform~ through a Ie~on or a 5Ufgicol shuntEvidence of periphllH"al vosculor dl.eoH dislontlo the PUrl(:tuf8 sileAcoogulopoltly or medillm- to h;gh.do~ anticoogulation theropy

Page 48: Arterial Blood Gases Made Easy - I. Hennessey, A. Japp (Churchill Livingstone, 2007) WW

ABG SAMPliNG TECHNIQUE

RADIAL ARTERY SAMPLING

• Pt:rform a modified Allen test to ensure adequab> ('oliMeralcirculation from ulnM arteryl (rigure 20).

• Position the p,ltknl's h<llld <.lS shown in Figure 19 with th~

wrist extt:uJeJ 20--30°. Greilter t:xtension of the wrist mayimpede arterial flow.

• Identify the radial artery by palpatinl'; the pulse; choose a sitewhere the pulse is prominent.

• C1~an the sampling site with an alcohol wipe.

• EXf*1 th.. heparin from syringe.

• Ste,ldy your hand on tile patient's hand, as showlI, th~n insertthe n~leat 45°, bevel facing up.

• Be surf> 10 insprt the need Ie slowly to minimise the risk ofarterial spasm.

• When the llt.wle is in the artery a flash of pulsatile blood willappear in the barrel of the needle. Most ABC syringes will th('tlfill under arterial pr"'SSllr(> (see info box over page).

• Obtain at least 3 UlL of blood before withdrawing.

'However, the value of routinely pedorming a modified Allen's test prior toarterial puncture has been questioned, in part due to its poor SP1lSitivity andspecificity for identifying inade.l' ,atE' col1aleral cin:ulalion. (Slogoff S, Kc-"tsAS, Arlund C. On the s.....fcty of cadi"l artL'rY (;;lJUlul"tiOIl. An"esthesioloj;\Y1983;59:42---47).

44

Page 49: Arterial Blood Gases Made Easy - I. Hennessey, A. Japp (Churchill Livingstone, 2007) WW

ABG SAMPLING TECHNIQUE

ngure 19 PO$jtioning 01 _i511ot A8G sompIing.

Arleriol sampling !porticvlor/y from the rodiol ortery) can be extremelypain/vI; dlscomforf can be reduced by inj.cring 1 ml 011 % lidocaine, at!he needle inMtrtion ~ite priOJ 10 wmpling

Venous _ &16.iaI lilac II?

Ooft. non.pul,otil. blood that requirti monuoI wction 10 os.pimle ofteninctocole$ a Y8OOt.I$~~ In _e modVcOl'dioc orr.g). AnoIherdve " when~ on A8G OldywS 's ''9n~1!y lower than 500: on pulseWtirnelry.

.,

Page 50: Arterial Blood Gases Made Easy - I. Hennessey, A. Japp (Churchill Livingstone, 2007) WW

ABG SAMPliNG TfCHNIQUE

AFTER SAMPLING

• Once duequatc blood has bft>n obtamed, remo,'c the needle andapply firm, d1rect pn.'S:iourc to the samplp ~lle for at least 5 minutes(and until bl~ing has cea:x'd).

• lliposc of all sharps and contammated matcrials approprialply.

• l'nsHrt> that no air bubbles are present in the samph~, as thcymay compromise results. Any sample with more than very fili~

bubbk--s should be discard<>:d.

• The sample Iohould be ;lnaly~ promptly: if the transit lime islikely 10 exl'ffd 10 minute'S, then the syringf' c;hould be blon.'\.! oncrushed ice,

• If sampling IS WlSu«cssfuJ it is often advbablc 10 repeat lhe teston the OPPOSite wnst ib e\'Cfl sliW11 irritation of the artery on the

first <IUempt may havp provoked arterial sp.15m, thwarting furtherattempt:. at puncture

..

I

Page 51: Arterial Blood Gases Made Easy - I. Hennessey, A. Japp (Churchill Livingstone, 2007) WW

,

ABG SAMPLING TECHNIQUE

InWuct h patieQ 10dlnctlliIitIIt fist !henoc:cllde bJltl;wlenes

- -'--LJ.

c

Rerllll .,. .... ar1efy.• fit ooIow IllUns to ..

r.1dMllW'l1Clllhishi ?m.., , I m:wtion

figure 20 M.ocJif,ed Allen test.

47

Page 52: Arterial Blood Gases Made Easy - I. Hennessey, A. Japp (Churchill Livingstone, 2007) WW

WHEN AND WHY IS ANABG REQUIRED?1. TO ESTABLISH A DIAGNOSIS

The- ABc. i~ integral to the diagnosis of respiTo1tnry failure andprimary hypt>n'pntilahon. It also u.lltlllifics the pn>sencp of metaboliclkidosis and alkalosis.

2. TO ASSESS ILLNESS SEVERITY

Thp four ABC valUel> l>hown in the t.1ble bl:>low, In aJditiun to owrallclinical QSHSSmm/, may help id\;~lIti{y critically \lnwpl1 patientsrt:y,uiring Llq~ent inteTVf>ntion. I

Pao, < 8 kPa

Bdow 8 "'0, fo{l~ in Pro, produce a marked redudion in Sox>, Ithe 'sleep'

port of 1M 0, dl$KIClOlion curvel and hence 0, content 01 bbod

Rising P0c02

Since .eool compem.ation occurs ewer dayJ 10 weeks, ocu~ riS&s in PocO]

prodoce 0 cOf.espooding drop in pH. In respiratory distress, rising POCO]often signifies exhaustion and 15 on omil'lOl./S s'gn. Polienls require Ulgen!

,e:ve!$01 of !he proefl~, Ieoding to ven"lotory foilure or auisted venhlahon

at: < -10/HC01 c 15-

This ....:rtu. is included in -..01 ~_...Iy 5COring syVelns arK!, when d""" klIodio: ocide»is, indicates 5e\Iere hypoluO of cenuktr IeYeI

H+ > 55/pH < 7.25

A ~i9llilicant decrease in pH ovl$ide the nOHoal ra"9fl indicates thatcompeosalory ~ir.ms have b.en overwhelmed and i$ a medical

emefgeoc:y

••

Page 53: Arterial Blood Gases Made Easy - I. Hennessey, A. Japp (Churchill Livingstone, 2007) WW

WHEN AND WHY IS AN ABG REQUIRED?

3. TO GUIDE AND MONITOR TREATMENT

Rf'&uliU" ABC momtoring can help provide early warning of

do;>lcriorabun and jl1d~e the cffe<:tivl;>ness of therapeutic inlprventiol\:>,It i:o essential for tilrating 0, therapy in patients with chronic tyl't! 2

rt'Spiratory f.lilurc and for optimising \'enhlator YffinKS.

Box 1.7.1 Clinkal scenorios In which an ABO is us.ful

Establishing dtagnosb ond aSMsstng i1lneu MYel ilt

5u~ hypercoptiio It PocoJDrowsiness, Roppmg Iremol', bounding pvI:IC~

Clinkol dete.iorotion in polient with chrOflic type 2 re$.pirOiOry

impoirm.nl Of' prediipO$ing condItion le.g. chronic ob~ud1Ve

pulmonory diseas.e\

Suspeded severe hypoxoemlO

Very low Of unrecordable 0, saturaTion; CYOrlO$i~ -Severe. prolonged or WOfSflning m,plrQlory distress

Smok.e Inhalotion Ica.boxyhuemoglobln level)

Hypervenlilonon (confirm IP~, check kw- UIldertying m.lOboIic ocidoslsl

At,. detel'lOl'ohon III consciousne!oS

Any leVu.ely unweH potient'

Pulso oximetry unreliable 01 $U~piclouS re~ull

As port of a 'A<:ogmsed Ilinen severity scoring ~y,tem le.g. Glosgow

cri...in in poncreohhsl-

Guiding II ealli"'" and ~itonng respon5e

Mechanically ventiloted JXItients

Potients receiVing non-Invosive o"i,ted vcntilolton

Patients w,1t. respiratory failure

Potlents with c:hroni<: hyperCQpmo receiving~

Ctiticolty ill polients undergoing surgery

Candidote, lor long-term olt)"96n the,apy

"Inc:koding, bu4 ,101 ."",1dIId lo, '''0(1. ~psi., buo n., mol'" 1raun'Ia. ocute obdomcn,poisCO'Iing, ~oocl;ocJrenol/hepo1i<: k>iIur.. JootJe••eaoocidosi.

.... .9

Page 54: Arterial Blood Gases Made Easy - I. Hennessey, A. Japp (Churchill Livingstone, 2007) WW

COMMON ABG VALUESThe followin/o; parameters arc commonly found on ABC reports andar", proviJ~ for wC",rence (normal ranges in btadcts):

W 135-45 nmol/ll < 35 _ olkoloemio, > 45 = ocidoemio

Concentration of free hydrogen ions: lhis is a measure of how addleor i1lkalinc a ~lution IS.

pH (7 35-7.45) < 7 35. ocidoen'llo, > 7.45 .. olkaloemio

Neg.1.tive 10K of the H' ion concentration: this is a commonrepresentiltion of th", H' I.VI'l(ltfltr..tion. Due to !.he logarilhmic natureuf the :Kale, !lOmaU changes in the pH actually represent largt' changesin the H- C'OJl("M\tration

f'O:ll> 10.6 kPo or > 80 mmHg in arterial bbod on room oirl

rdrtial pres:;;ure of~: can be thought of as~ drive for 0:1: mokculcsto move from onE' pl3cp tn anothM'. I'O:t L'l not a measure of OzCOfltent tmt it does dett'flJline the extent to which Hb is sahJratedwith~.p~ refers specifically 10 the partial pressure of Oz in arlerWblood.

~ 14 7-60 kpo Of 35-45 mrnHg in arterial blood)

Partial pressure ofC~: can be lhoughl of as the drive for Co,;molecuI~ to muve from~ place to another and (unlike Po;) isdirectly proportional to the amount ofC~ m blood. pOCO:! refersspecifiCAlly to the partiaJ pressure of~ in arlmlll blood.

5o:l1> 96% on room oir)

0 1 saturation of haemoglobin: the~tage of o,.-binding site>on lib proteins occupied by~ molecules. This is a measure of howmuch of lhc blood's ~-earryingcapacity is being used. Sao-l refers~ifical1y to the 0" saturation of rJrtmol blood.

,.

Page 55: Arterial Blood Gases Made Easy - I. Hennessey, A. Japp (Churchill Livingstone, 2007) WW

COMMON VALUES AND THEIR MEANINGS

HCOJocl (22-28 rnmol/ll

Actual bicarbonate: the plasma bicarbon.:ttc concentratiun caiclilutcdfrom the actual pe02 allu pH JIle<l::;u~mentsin the arterIal bl(l()(i,~mple. High bicarbonate level" "ignify a mptaholk alkalosis and lowIt'vli'ls signify a metabolic acidosis.

HC03$1 (22-28 mmol/l)

Standard bicarbonate: thp plasma hki1rbonate concentrationcalculatedjrom the Pe02 and pH measurements in the arteriill bloodsample after the pC02 in the sample has been corrected to 5.3 KPa(40 mmHg), The authors recommend using this mpaSlll"(>ment ofhirarh0natp in A Kt--; analysis.

BE (-2 10 +2l

Base exre::;s; a calculation of the amount of base that nP1:'ds to be

added to, or removed from, a sample of blood to achieve a neutralpH, at 37°, after PeOz/las been cortt'Cled 10 5.3 KPa (40 mmHg), Apositive BE inJicates that there i::; more base than normal (metabolicalkalosis) and a negative BE mdicates that there is l~ss base thannormal (ml2'tabolic acidosis).

ladate (0.4-1.5 mmol/l)

An indirect measure of lactic acid: high levels of lactic arid are a signof tissue hypoxia.

Hb n3-18 g/dl men, 11.5-16 g/dl women)

Pla,5ma hapmoglobin concentration: this effectively determinesblood's capacity to carry O2 ,

Na 1135-145 mmol/LJ Plasma sodium conc~ntration,

K(3.5-5 mmol/ll Plasma potassium concentration.

Cl WS-:-l 05 mmalll) Plasma chlorid~concentration.

iCa {I.(}-l .25 mmol/l) Plasma ionised calciwn concentration.

Glucose (3.5-5.5 mmol/l if fosting) Plasma glucose concentration.50

Page 56: Arterial Blood Gases Made Easy - I. Hennessey, A. Japp (Churchill Livingstone, 2007) WW

MAKING ABG INTERPRETATION EASYThe golden rule for making ABC interpretation easy is Tobie 1.9.1 An.Sling hypoxaemia ieverityto assess pulmonary gas exchange and acid base statusindependently. p~, Sao,

Mild 8-10.6 IJ'Q 90-94%

"AS$,n~G:"PlLQiiO~4YqA'S .!'~Il~i'J~ 60-79 mmHg

• Using the algorithm opposite, classify gas exchange Moderate 5.3-7.9 kPa 75-89%

into one of the four possible categories.40-59 mmHg

• If there is type 1 respirator], impairment, assessSevere < 5.3 kPc < 75%

< 40 mmHgseverity of hypoxaernia (box 1) or: High fiO~ requirements

• If there is type 2 respiratory impairment, establish to maintain odeqoote Poo~

whether it is chronic or acute, then assess severity of0~" n~f~;hypercapnia and hypoxaemia (boxes 1 & 2) ;~~)'''11 . "'~" ,'".- ' ectp, •. -, .- .. " .. ,,< .", ..

• If the (<llegor)' is hyperventilation. determine whether Severity is nol reloleo:::! 10 abr.olule Po::~ "ollie butil is primary or secondary. 10 Ihe raie of PacOj rise and degree of blood pH

derongemllnt (pH < 7.25 • $evere acidaemia). The

presenctl of exi>a ...stion is olso on o'llinous sign.

Page 57: Arterial Blood Gases Made Easy - I. Hennessey, A. Japp (Churchill Livingstone, 2007) WW

MA

KIN

GA

BG

INTERPRETATIONEA

SY

~

__~

_.::..'i-

..I

"

Page 58: Arterial Blood Gases Made Easy - I. Hennessey, A. Japp (Churchill Livingstone, 2007) WW

INTERPRETING ACID·IASE STATUS

-,­PlCD,- 4,7-U kPiI

or ~5mm1-1g

BEj-llOo2j

~11 (22-28~

pH (7.35-7 45)

If' (35-45 Md4)

••• Usc the flow chart opposite to broadl}' classify acid-base status.• If the patient has a metabolic acidosis, calculate the anion gap to narrow dov.'Tl the differential diagnosis.

Anion Gap"" (Na + K) - (el + HC03) Normal =10 - 18mmol/l

• U the precise acid-base derangement is not immOOiately clear (ego middle column) then remember thefollowmg points:

Always consider the clinical conlexl when interpretingacid-base status.Metabolic compensation takes days 10 occur, re>piratorycompensaoon takes minutes.Overcompensation does not occur.An apparent compensatory response could repl"e5ent an opposingprimary process.

• Note that with a very mild acidaemia or alkalacmia both PaC02 andHCO) may be just within the "normal range" (considered to be a mIldmixed aad-base dISturbance).

~~z..~>§~

Page 59: Arterial Blood Gases Made Easy - I. Hennessey, A. Japp (Churchill Livingstone, 2007) WW

MA

KIN

GA

!GIN

TERPR

ETATION

EASY

••

Page 60: Arterial Blood Gases Made Easy - I. Hennessey, A. Japp (Churchill Livingstone, 2007) WW

APPENDIX 1

The A-o erodi«llillhe difference Dtt.o.een !he~ in oNeoIi IPAo"l ondthe Po, in ortef"ioI biood lPa011. p~ is meowred Of! A6G bur PAc" hos

to be calculated us.ng !be oIveoIor gc» equation 1- box beIowj.

A - G flruclien' • f1'~ - ,..."

1111 noflnolly bs thon 2.6 i.:Po (20 mmHg). oIIhough it increo~with oge

and F~, This meam Ihat:I. the l'lOl'moI range fOf Poo, IoUs wilh oge2 me A-o grodient is most accurate when performed on room air

PAo, (kPo) = (1;0, • 93.1) - (Poco, • 1.2)

~

Neo:2 ImmHg) = l'iD, • 713) - (I'ato, .. 1.2)

5.

Page 61: Arterial Blood Gases Made Easy - I. Hennessey, A. Japp (Churchill Livingstone, 2007) WW
Page 62: Arterial Blood Gases Made Easy - I. Hennessey, A. Japp (Churchill Livingstone, 2007) WW

THE ABG IN PRACTICE

CASE 1

History

A 25-ye-ar-01d man, with no significant past medical ~toI)',presents to the emergency de-partment with a 2-day history of fever.productive rough and woT5elling breathlessness.

Examination

He is hot and flushed with a temperature of39.3"C. He does notappear distressed but is using accessory muscles of respiration.There is diminished chest expansion on the left with dullness topercussion, bronchial breathing and coarse crackles in the left lowerzone posteriorly.

""'"lte:5pirotory role

BIoodpn=~

~ lroom airj

58

HM beats/min

28 breaths/min

118/70 mmHg

"""

Page 63: Arterial Blood Gases Made Easy - I. Hennessey, A. Japp (Churchill Livingstone, 2007) WW

Arleriol blood go~ On room air23/7/2006Unit no,: 00654545ID- John Simpson No<~1

W 31.8 nmol/L {35-451

pH 7.50 (735-745}

1'00, 3.74 kPo (47-6·0128.1 IT\rTlHg (3545)

Po, 768lcPo 1> 10.6}57.8 mmHg t> IlOl..... 23.9 mmol/l (22 28}

" -0,5 ml"lOi/l 1-2_2)

SPa, 88.7~ 1>9n)locto.. 12 10.4-1 5}

K 37 mmc1/l {3.5-51No 138 mlTlOljl 1135-145}CI 99 mmol/l (95-105}

ICO' 1.2 mmol/l {l-1.25)

Hb 15g/dl [13-18)

Glucose 5.4 mrnol/l (3,5-5.5]

Questions

1. a) Describe his gas exchange.b) Describe his acid-base status.

L Should the patient receive supplemental O:z?

J. Is pulse oximetry a suitable alternative to repeated ABGmOnitoring in this case?

,.

Page 64: Arterial Blood Gases Made Easy - I. Hennessey, A. Japp (Churchill Livingstone, 2007) WW

THE A8G IN PRACTICE

CASE 2

History

1\ 34-ye,1f-old morbidly obese female with a body mass index of 49has an ABC sample taken as part of her prroperatiYe assessment forweight reduction surgery.

Apart from morbid obesity and type 2 diabetes, she is otherwise w

and has no ~piratory symptoms.

60

Page 65: Arterial Blood Gases Made Easy - I. Hennessey, A. Japp (Churchill Livingstone, 2007) WW

Al1eriol bbad 901 Gnat.-14/01/2005Uril no.: 553Q208e

10: Morcella P1antogenet

""'malH" 45 nmol/l 135-451pH 7.35 [7.35-7.45)

Ito, 7.3lPo (4J-6.0I54.8 mmHg (3'-45}9.6 i:.Po I> 10.6172.2 mmHg I> 80)

Bicarb 29 mmol/l. (22-281

" .,).8 1-2-+2)

Sib, 96" (> 98'%)

'odaM I (0.4 - 1.5], 4.7 mmol/l 13.5-5)No 134 mmol/l 1135-1451cr 102 mmoVl 19S-105Ju· , 2 mmol/t 11-1.25)Hb 13g/d1. (13-18j

GIl)cXlSEl 9 mmol!l 13.5-5.51

Questions

t. a) Describe her gas exchange.b) Describe her acid-base status.

2. "''hat is the most likely diagnosis?

THE A8G IN PRACilCE

61

Page 66: Arterial Blood Gases Made Easy - I. Hennessey, A. Japp (Churchill Livingstone, 2007) WW

THE ABO IN PRACTICE

CASE 3

History

A 24-year-old female nursing student attends hospital complainingof sudden-onset breathlessness. She flew to the UK from Australiathe previous day and is very concerned she may have a pulmonaryembolism. She has no pleuritic pain, haemopt}'sis or leg swelling.no past hisrory of lung disease or deep-vein thrombosis and is anon-smoker.

Examination

She appears anxious and distressed. Her respiratory rate is elevatedbut eMt examination is unremarkable and there are no clinical siof deep-vein thrombosis. A chest X-ray reveals no abnonnalities.

p,,~

Respirotory rote

B600d preuure

~ lroomotrj

62

8e beals/min

22 breaths/min

124/76 rflmHg

95"

Page 67: Arterial Blood Gases Made Easy - I. Hennessey, A. Japp (Churchill Livingstone, 2007) WW

23f7/2006u"" no.: 00654.54510: Jell Royds

THE ABG IN PRACTICE

eopH

"'o,

Bicorb

BE5po,lodoM,No

d

J l.2 nmol/l

7513.90 kf'iJ293 m."Hg

10.3 kilo77.0mmHg

25.0 mmoVt+0.7 mmol/l93.7%

1.04.3 rnmoI{t

141 m<nol/l101 mmol/l.1,2 mmol/l

13 a/dL4.6 mmol/l

Normal

135-451(7.35-7.45)

1A.7-¢·0I135-451f> 10.61(> 801{22-28)

1-2-+2,(> 98%)

(0." - 1.5113.5--51fl35-145)(95-1051{1-1.251(l3-1BI(3.5-5-5)

QueSt101U

1. a) Describe her gas exchange.b) Describe her actd-base status.

2. \\'hat is the likeliest diagnosis?

Page 68: Arterial Blood Gases Made Easy - I. Hennessey, A. Japp (Churchill Livingstone, 2007) WW

THE A8G IN PRACTICE

CASE 4

History

A 78-year-old male on a surgical ward is found unresponsi\'e,having returned, a few hours previously, from a complicated opencholecysteclomv.

Re... iew of his charts J'e\'eals he has recetved three 10 mg injectionsmorphine since returning to the ward, in addition to the morphinedelivered by his patient-controUed analgesia device.

Examination

The patient is unresponsive with ~llow respirations and bilateralpinpoint ptJpils.

PlIlsll role

Respirolorl' role

8Iood preullle

500,8M

..

90 beob/min5 breatlu/mirl

98/64 mmHg.... i5.6 mmol/l

u

Page 69: Arterial Blood Gases Made Easy - I. Hennessey, A. Japp (Churchill Livingstone, 2007) WW

,Ivftwel blood gos On 28%0"

18/09/2006l}r1it (lCl.: 6799986[l Henry Spc~~

Normel

". 65.4 nmcl/l {35-45/

pH 7.18 (7.35-7.45)

"'" B.2 ~Pe (47...o.0J62 mmHg {35-451

"" 11.76 kJ'e I> 10 6187 mmHg [> SOl

Bicarb 22.4 mmoVL [22-28)

" -1.5 mmol/l 1-2-+21

"'" 99.8~ (> 98'l.j

~-1 10..4-1 5), 4.4 mmol/l p.5-5J

No 137 mmol/l 1135-1451

a 103 mmoI/L (95-1051;Co' 1.16 mmeVl 11-1.251

'*' 11 g/dl !I3-181Glucose 3.9 mmol/L (3.5-5.5)

Questions

1. a) De$(:ribe his gas exchange.b) Describe his acid-base status.

t What IS the m06t Wc:ely diagnosis?

1 What treatment does this patient require?

THE ABG IN PRACTlCE

Page 70: Arterial Blood Gases Made Easy - I. Hennessey, A. Japp (Churchill Livingstone, 2007) WW

ThE ABG IN PRACTICE

CASE 5

History

A 75-year-old man IS brought into the emergency department by hisfamily. He is extremely short of breath and struggling to speak.

Following a conversation wilh his family it emerges that he has along history of chronic obstructi\'C pulmonary disease.

o.'Cf the lasl 3 day'S his breathing has worsened consideTably and hehas expectorated increased \'oIumes of sputum.

Examination

The patient is struggling for breath and appears extremely dis,lre".He exhibits signs of chest h)-pcrinflation and is breathing throughpursed lips. Breath sounds are generally diminished but there areadded sounds.

PulMiRe~rak:Hy JU'tI

8Iaod preuure

Temperature

Sa02%

120 beals/min

26 bnlolls/min150/80 ll'ImHg

36·C

'I"

..

Page 71: Arterial Blood Gases Made Easy - I. Hennessey, A. Japp (Churchill Livingstone, 2007) WW

Arterial blood gas On oir10/07/2005Unit no.: 77655349

ID: Joseph Spielmol1Norma!

H' 39.5 tlmol/l (35-45)

pH 7.40 [7.35-7.45)

Pto" 4.9 kPo [4.7-6·0136 mrnHg (35-45)

Po, 5.6 kPo [;. 10.6J44 mmHg [> aOJ

Bicorb 23 mmol/l [22-2a}

BE -1.2 mmol/l {-2-+21

SPO, 80% I;. 98%)lactate 1.0 {O.4-1.5]

K 4.1 mmol!l (3.5--5)

No 137 mmol/l (135--145J

CI 99 mmol/l {95~105]

iCa' 1.1 mmoljl (1-1.25)

Hb 16.5g/dl (13-18)

Glxose 3.8 mmol/l (J.N.51

Que5tions

1. a) Describe his gas exchange.b) Describe his acid-base status.

2. Should you provide him with oxygen?

THE A8G IN PRACTICE

.7

Page 72: Arterial Blood Gases Made Easy - I. Hennessey, A. Japp (Churchill Livingstone, 2007) WW

THE ABO IN PRJI.cllCE

CASt 6

History

The pabent from the pre"ious case is treated in the emergencydepartment with nebuliscd bronchodibtors, oral prednisolone andantibiotics. He is then transfened to a respiratory ward where he itadministered 28% oxygen by a fixed concentration mask. Despitethis, his Sa02 (as measured by pulse oximetry) increases onlymarginally and there is no imprO\'ement in his symptoms.

Examination

Examination findings in the chest are unchanged but he now appooolemausted and slightly confused.

--- -- ---Pulle

Re~loIory mle

8bxl pressure

SoO,Temp.... alure

120 beats/min16 breath~/mln

120/80mmHg83'X.1on 28% Q,]Moe-

A repe.1t ABG is performed (6 hours after the first ABG).

"

Page 73: Arterial Blood Gases Made Easy - I. Hennessey, A. Japp (Churchill Livingstone, 2007) WW

,ArMori'" b£ood gas On 28% OJ11/07/2005Unit no.: 77655349

10 joseph Spielman

N~ma1

W 51.4 rwnoI/l 13>-<15)

oM 7.29 17·35-7.451

"'" 6.9 kPo 14.7~·0l

52 mmHg 13>-<15}

II>, 6..4 kPo {> 10.61

48 IfImHg (> 80)

-'" 24 mmot!1. f22-281

" -0.9 mnol/l 1-2-+2}

511>, .... I> 98'%)lactate 1.0 (0.4-1.51

K 4.0 mmol/l (3·5-51No 137 mmol/l. (135-I45J

d 99 mmai/l (95-105)

"". 1.1 mmol/L 11-1.25)... 16.5 g/dl 113-181Gtuco~e 4.2 mmol/l 13.5-5.5}

Questions

l.a} Describe his gas exchange.b) Describe his acid--bi'lsc ~t"tus.

2. Should his oxygen now be stopped?

THE ABO IN PRACTICE

••

Page 74: Arterial Blood Gases Made Easy - I. Hennessey, A. Japp (Churchill Livingstone, 2007) WW

THE ABG IN PRAcnc:e

CASE 7

History

A 77-year-old woman is admitted to the stroke ward with right-sidweakness, visual disturbance and slurred speech. She is commencedon nasogastric feeding due to swalJO\\ing problems but has a largevornlt 24 hours later. She initially appeal'S well but oyer the next fewhours develops .....orsening breathing difficulties.

Examination

She is agitated, distressed and pyrexial. A dull percussion note andcoarse crackles are evident at both lung bases. Other than acuteconfusion, neuroIogkal findings are unchanged from admission.

Pul$eRe$j>irolory ro)e

Blood preUl,,Ire

SaO, '''''' 0,)

92 beob/mln28 breotlu/min112/65 mmHg.,,,

70

Page 75: Arterial Blood Gases Made Easy - I. Hennessey, A. Japp (Churchill Livingstone, 2007) WW

·Merial blood go$ On 60"4 O2

23/7/2006Uril no.: 00654545II). Mrory Woteo

No"""W 38.8 nlrlOllt f.l5-45Jpi< 7.41 (7.35-7.451

"'..0, 4.43 lJ'o (4.7-<10)

33.2 mmHg 135-451Ib, 8.67 kpo (> 10.6)

65.0 mmHg (> 80)

Bico,b 21,2 mmol/l [22-281

" -2.8 mmol/L 1-2-+2)SAo, 927% I> 98%1loctcle 1.6 (OA-I.5)

• 4.0 mmoi/L 13.5-61No 144 mmoVL (135-145)

C 103 mmollt (95-1051iCo· 1.2 mmol/L 11-1.25)

Hb 13 g/d!. (13-18)

G1"'COH 6.6 mmol/L [35-5.51

Questions

1. ,I) De;eribe her gas exchange.b) De;eribe her acid-base status.

2. What is the most likely diagnosis?

3. ~ her condition mild, moderate or se...ere?

THE ABG IN PRACTICE

71

Page 76: Arterial Blood Gases Made Easy - I. Hennessey, A. Japp (Churchill Livingstone, 2007) WW

THE AEIG IN PRACTICE

CASE 8

,96 beob/min24 breotbs/min13B/82 mmHg78%

""~ResptrolOry role

Blood prllmH8

SaO, (100Il'I oir)

Examination

He is lucid, alert and nuJdly distressed. He is using acnsscrymuscles of respiration and breathing through pursed lips. Chestexamination reveals features of hyperinlliition, generally dimin'breath sounds and scattered rhonchi. (wheeze).

History

A 68-yt'ilr-Qld man with chronic obstructive pulmonary disease isreferred 10 hospital by hIS doctQr with a short hislory of increasedbreathJeossness and noduced effort tolerance. He is nonnall.}' caof walking around 500 metres but now has difficulty dressing abreathless at rest.

Page 77: Arterial Blood Gases Made Easy - I. Hennessey, A. Japp (Churchill Livingstone, 2007) WW

THE ABO IN PRACTICE

Arterial blood gos On air

23(7/2006Ullil 110.: 00654545

tD: Hamish RoyNormol

W 43.2 nmoljl 135-451

pH 7.36 (7.35-7.451

Pco, 7.20 kPo (4.7-6.D)

54.1 mmHg 135-451

Po, 5.3 kPo I> 10.6)

40 mmHg (> 80)

8icarb 30.6 mmolfl (22-28]

BE +4.9 mrooI/l (-2-+2)

SPo, 75.2% (> 98i~)

(octale 1.2 [0.4-1.5)

K 3.7 mmol/l 13·5-5)

No 144 mmolfl {I 35-145)

CI 102 mrnol/L 195-105)

iCo' J.2 mmol/l [1-1.25)

Hb J6g/dt {\3-18)Glucose 4.9 mmol/l 13·5-5.5)

Questions

1.3) [)es(ribe his gas exchange.b) Describe his acid--base status.

20 Which one of the foUowmg ABC values is most likely to havechanged significantly in the past 24 hours; pH, PC0l-- 1'0:2 orHeO)?

3. \'I,1tich two of the above ABC values indicate the need for cautionwhen providing O2 therapy?

73

Page 78: Arterial Blood Gases Made Easy - I. Hennessey, A. Japp (Churchill Livingstone, 2007) WW

THE ABG IN PRACTICE

CASf9

HistoryThe patient from the previOlis case is treated with nebuliscdbronchodilators, oral prednisolone and 6O'Y.. 0: by face mask. Hisoxygen saturations improve significantly but when he is revie\>ved1 hour later, his condition has deteriorated <Iud he is un.abJe to

provide a history.

Examination

He is drowsy and barely rousable. He no longer appears to be inrespiratory distress and his respiratory rate has fallen to 14breaths/min. Olest examination is unchanged from pre,'iously.

p~~

Il:e$pirotory rote

Blood ........~ (room air)

88 beoh;/minI. breaths/mm132/80 mr'IHg96..

,.

Page 79: Arterial Blood Gases Made Easy - I. Hennessey, A. Japp (Churchill Livingstone, 2007) WW

THE ABG IN PRACOCE

Arferiol blood gas 0..60%0,23/7/2006\,kIit 00.: 00654545

ID Homish RoyNonool

... 50.8 nmollt 135-45}

pH 7.29 {l.35-7.45)

I<D, 8.7 lrJ'o (4.7-6,0)

65.3 mmHg (35--451

""11.2 \'Pc I> 10.6}

84.0 tr'IllIHg t> 80)

"""b 30.3 mrnol/l. f2'-"0I

" ....7 mtne>1/l ....2-+21

SIb, 96.2~ f> 98'%1

"""'" I 2 10.4--1.51

K 3,6 mmol/l {3.5-51

No 14.4 mrnol/l (135--1.45)

CI 102 mmol/l (95-1051

iCa' 1.2 mmol/1 (1-1.251

Hb 16 g/dl 11J.-18)(;1,- 5.0 mrnol/l (3.5--5.51

Questions

1. a) Dl-scri.be his gas exchange.b) Describe his add--base status.

2. What has been thecausc of his deterioration?

Page 80: Arterial Blood Gases Made Easy - I. Hennessey, A. Japp (Churchill Livingstone, 2007) WW

THE ABG IN PRACTiCE

CASE 10

History

A 21-year-old woman presents to the emergency department witha 6-hour history of worsening breathlessness and wheeze. She hasa past history of asthma with two previous exacerbations requiringhospital admission. She now feels very breathless and is obtainingrelief from her salbutamol inhaler.

Examination

She is tachypnoeic at 30 breaths!min, is using her accessory musclesof respiration and only just managing to speak in full sentences.

Auscultation of her chest reveals widespread polyphonic wheeze.

Pul~e

Blood pressure

SrD,Peak elCpiralar)' flow

115 beats/min120!80 mmHg96% lroom airj

160 LIs lpredicted. 400 lis)

76

Page 81: Arterial Blood Gases Made Easy - I. Hennessey, A. Japp (Churchill Livingstone, 2007) WW

THE ABG IN PRACTICE

,Artenol blood gas OnD"12/11/2005U,nl no.. 126399...3;0 Jessica Gokhon

"""moltr 42 nmoI/L 135-45)pH 7.38 17.35-7..45)

"'''' 5.8 U>o (....7-6.01

43~Hg 135-451FO, 10.2 kPo I> 10.61

76 mmHg I> 80)BI~orb 24 mmollL 122-281BE -1.3 mmol/l 1-2-21500, 96" I> 98%1

lado" 1 10.4 - 1.51, 4.0 mmol/l (3.5-51~o 140 mmol/l. 1135-1.451a 99 mmol/l (95-1051

I ,Co' I 12 ",maI/l 11-1251Hb 130 g/dL P 1.5-15·.51Glucose 5 mmol/l (3 5-5.51

Questions

1. a) Describe her gas exchange.b) Describe her add-base status.

2. Which of the above ABC values gives the greatest cause forconcern?

3. flow would ,·ou classify the seH''I"ity of this asthma attack?

77

Page 82: Arterial Blood Gases Made Easy - I. Hennessey, A. Japp (Churchill Livingstone, 2007) WW

THE ABG IN PRACTICE

CASE 11

History

A 23-year-old woman presents to the emergency department willi ..painful ankle, following a simple trip. While being examined by theemergency department doctor she becomes extremely agitated andupset. Despite a nonnal ankle X-ray and extensive reassurance bythe emergency departm€I1t staff, she refuses to believe that her anis not broken and staTts crying. V\-1l.i1e leaving the d('partment, shedevelops a dutching: sensation in her chest, shortness of breath anda tingling sensation in her hands and around her mouth. She ll.:'pOthat she feels unable to take a deep breath.

Examination

The patient appears frightened and extr\;.'tllely distn>sscd. Otherthan t1lchypnoea and a mild sinus tachycardia, cardiorespiratoryexamination is unremarkable. Electrocardiogram, chest X-ray andpeak tlow measurements are all normal.

Pulr.eI<:es.pirotory rateBlood pressure

s,D,

7.

96 beats/min36 breaths/min130/BO mmHg

100%

Page 83: Arterial Blood Gases Made Easy - I. Hennessey, A. Japp (Churchill Livingstone, 2007) WW

Medal bk>od gas Oneir

12/11/2006l)roil no.: 12534943

!O Tril'lllY Forqhuof

No<mal... 29 nmolfL (35-45)

pH 7.53 {7.35-1.451

Ito, 3.1HPo (.4 7-6.01

24 mmHg 135-45)

Ib, 1".3 ~Po I> 10.61108 mmHg I> BO)

Bicorb 24 mmol/L 122-2BI

BE -1.8 mmol/L 1-2-+21500, 99% (> 98%)

eoc- 1 10·4-1.51

< 3.5 mmol/L 13.5-5)No 140 mmol/l 1135-145}CI 99 mmol/L (95-105)

iCo' 0.9 mmoJjt (1-1.251

"b 12·°9/& (I1.5-15.5)

"'OC~ 5 ntmol/l 13.5-55)

Questions

1. a) Describe her gas exchange.b) Describe her ucid-base status.

2. Are there any other abnormalities?

3. What is the likely diagn~is?

THE ABG IN PRACTICE

79

Page 84: Arterial Blood Gases Made Easy - I. Hennessey, A. Japp (Churchill Livingstone, 2007) WW

THE A8G IN PiACTlCE

CASE 12

History

A 4O-Yl'ar-old male is pulled from a house fire and brought 10 theemergency dt-partment in the early hours of the morning:. Thepilrillnedics estim."lte he is likely to have been trapped in asn\oke-filk--d room for up to 20 minutes prior to rescue.

Examination

The patient is heavily contaminated with soot and smells stronglyof smoke, Fortunately he has not sustamed any thermaj injuries, Heappears to be confused and has just vomIted.

Baseline observations are nonnal, with an oxygen saturation of 99%on 15 L Oz by rn."l.<;k.

80

Page 85: Arterial Blood Gases Made Easy - I. Hennessey, A. Japp (Churchill Livingstone, 2007) WW

THE ABG IN PRACTICE

Arterial blood gos On 15 L O2 by mask

10/08/2005

U'lIIlO.: 77634566

1D: Robert JooesNormal

W 414 nmol!l (35-4.51

pH 7.36 (7.35-745)

",0, 4.5 kPn (47-6.0)

)4 mmHg (35-45)

47 "'" I> 10.61353 mmHg I> 801

Bicorb 18 mmol/l (22-281

BE -5.5 mmol/l {-2--t21

51'>, low. (> 98'%}

tac!l:rle 2 (0.4-1.51

K 3.0 mmol/l p.5-5)

No 145 mmol/l (135-1451

CI 103 mmoI/l {95-105)

0C0' 1 1 mnoI/l (1-1.25;

Hb 14 9/& {lJ-1SI

Glucose .4 mmoi/l 13.5-5.51

Carbon monoxide G5SCIy

CO 40% [non-smokers < 3%1

{smokers < 10%1

Questions

1. ,1) Describe his gas exchange.b) Describe hlS acid---base status.

2. \Vhat is the mosllikcly diagnosis?

l. Which of the values pro\"ided isfDls.·fy high: POlOz,~ or Hb?

&l

Page 86: Arterial Blood Gases Made Easy - I. Hennessey, A. Japp (Churchill Livingstone, 2007) WW

THE A8G IN PRACTICE

CASE 13

History

A 79-year-old female has just been adm..illed to the general surgicalward to have a large bowel tumour surgW:ally remon!d.

The tumour \v...s discovered at colonoscopy after she presented 10 her.doctor with a six month history of rectal bleeding.

On admission she appearroto bese...erely short of breath andextremely tired. Further questiomng reveals thai her rectal blood losShas been no greater than usual.

---------~Pul$e

alood pre~$l,Ire

Re$piro/Qry rate

S002% (on air)

"

100 betti/min100/80 nwnHg24 breoiN./m,n100%--

JI

-

Page 87: Arterial Blood Gases Made Easy - I. Hennessey, A. Japp (Churchill Livingstone, 2007) WW

THE A8G IN Pl<:ACTtCE

Arterial blood gas On air06/06/2005Ullit no 94J778fD: Ethyl Swoiluon

NormolH' 32.3 nlToI/l 135-45}pH 7_49 17·J5-7.A5)Pco, 3_31 kPo (4.7-6.0)

25 mmHg {35-45111.9.Pa l" 10.6)89 mmHg I" 801

BiCarb 22rrmol/l. 122-2B}

8' -2 mmoI/l 1-2-+2)

5"" 99.8% (,. 98%),",Ia~ 1 (0.4-1.5)K J.B mmol/l (35-51No 138 mmol/l fl35-J.45)CI 96 mmol!l 195-1051,Co· 1.17 ll'I",oIA (1-1251Hb 6.8 g/dI. (13-181Gluco5e 3.9 mmol/l (3.5-5,51

Questions

1. a) Describe her gas exchange?b) Describe her acid-base stale':>

2. What is the most likely cause of her breathlessness?

3. What would be the most effe...·th,e way of improving~ delivery10 her li.%u('S?

83

Page 88: Arterial Blood Gases Made Easy - I. Hennessey, A. Japp (Churchill Livingstone, 2007) WW

THE ABC IN PRACTICE

CASE 14

History

A 78-year-old woman is admilled to the emergency department witsudden-onset, g('fll."ralised, severe. abdominal pain. She describesthe pain as being colicky with no particular f<ldiation. S~ does notcomplam o( any altef<ltion in her bowel hablt and hac; not vomited.Her only medical history is that of atrial fibrillation, (or which shetakes aspirin and digoxin.

EJlaminotion

On examination, the patient is h..x'lTlod~"l1"mically stable with warwell-perfused peripheries. Despite severe abdominal discomfort,abdominill examination is relatively unremarkable: the abdomen15 soft in illi (our quadrants and is only tender on deep palpation.No hernias or aneurysms aT\' palp.1ble i100 l'('Ctal exammation isunremarkable.

An abdominal and erect chest X-ray are taken and found to benormal.

During the course of the examination her dinical conditiondeteriorates and she is mon"j to the resuscitation area.

••

Page 89: Arterial Blood Gases Made Easy - I. Hennessey, A. Japp (Churchill Livingstone, 2007) WW

THE ABG IN PRACTICE

Arrreriol blood gas On 10 L O2 by mask10/08/2006Umlno.:n3421110: Su-. Ultik

Nonnal

H' 525nmal/l (3,S.....4S1

pH 7,28 fl·35-7.4S}Pc", 4.39 kPo IA.7-6,Oj

33 mmHg (35-45128.6 Jd'o I> 10.6}

215 mmHg I> 8018icarb 16.2 mmoI/l (22-2.)8E -10.4 mrnol/l {-2-+2)

5"" 99.8% (> 98%1

Lodo1e 3.2 10.4-1.5)

K 4.6 mmol/l 13.5-5)No 135 mmol/l 1135-1451CI 96 mmol/l. [95-I05}iCo' 1.16 mmol/l {I-US]Hb 12 g/dl 113-181Gloco.e 3.8 mmol/l (3.5-.5.51

Questions

1. a) Describe her gas exchange.b' Describe her acid-base status.

2. What is the most likely diagnosis?

IS

Page 90: Arterial Blood Gases Made Easy - I. Hennessey, A. Japp (Churchill Livingstone, 2007) WW

THE ABG IN PRACTICE

CASE lS

History

A 35-year-old woman with type 1 diabetes is brought to theemergency department by ambulance after being found in her houseverely unwell. Following a discussion with her parlner it emerg(>'S~ has not been eating for the past fe-.\' days due to a vomitingillness and, as a precaution. has also been omitting her insulin.

Examination

On examination, she appears drowsy and peripherally shut-down,with very dry mucous membranes. Her breath smells of acetone anher respirations are deep and sighing.

IM"Blood preswf.

Resplfatory fOIe

!~ (blood glLlOO5eI

130 beob/min

100/60 mmHg26 breaths/min

:>- 25 mmol/l

Ph}"Sical examination of her chest and ..bdomen is unremarkable.

••

Page 91: Arterial Blood Gases Made Easy - I. Hennessey, A. Japp (Churchill Livingstone, 2007) WW

THE ABG IN PRACTICE

Arterial blood 9Q$ On 10 L~ by mask27/02/2005Unit 1'10•. 7773S56610: IlJa Tonner

NormalH' 88.9 nmol/L 135-451pH 7.05 {7.35--7 ....S}-. I 5 !.Po (4.7-6·01

II ml'llHg 135-45128.4 kYo I> 10.6)187 mmHg I> aO)

Bicarb 6.0 mmoI/l 12.4-30JBE -25.2 mmoI/l 1-2_2}

SA>, 99."" (> 98%1

""- I (0.4-1.5)

K 4.6 mmol/l IU-51No 141 ",moI/l (135-14.51C! 96 mmol/l (9S-105JjCo~ 1.25 mrno!/l {I-1.25JHb 12 g/dl 113-18)

Gi...:;o~ 35 mmoI/l (3.5-5.51

Questions

1. a) Describe her gas exchange.bl Describe her acid-b."\Se status.

2. Calculate the anion gap

3. What is the m05t likely diagnosis?

.7

Page 92: Arterial Blood Gases Made Easy - I. Hennessey, A. Japp (Churchill Livingstone, 2007) WW

THE ABG IN PRACTICE

CASE 16

History

A 37-year-old well-known local vagrant is brought into thedepartment unconscious. He was fowld near a bottle of vodka andhalf-empt)' bottle of what appears to be methanol. It is unclear if hehas drunk any of the contents.

Exomination

The patient is unkempt with a significantly reduced consciouslevel (Glasgow coma scale score:= 9). There are no apparent focalneurological abnormalities.

II

Page 93: Arterial Blood Gases Made Easy - I. Hennessey, A. Japp (Churchill Livingstone, 2007) WW

THE ABG IN PRACTICE

Arteriol blood gos On air

10/07/2006Unit rlO,: 35477899

ID: Gary Sooness

Normol

W 63.3 rlmol/l (35~451

pH 7,20 (7.35~7.45)

Pco, 3.3 kPa {4.7---6.0]25 mmHg (35--4':::')

12.8 U'a (> 10.6)96 mmHg (> 80)

Bicorb 9.5 mmol/l (22-28)BE -16,2 mmol/l (-2...-+2)Sit>, 97.8% (> 98%)lactate 1.3 (0,4..-1.5)

K 4.5 mmotjL (3,5-51No 136 mmol/l (135-145)(I 99 mmol/l (95-1051iCo' \.1 mmol/L fl-L25]Hb 13.5 g/dl 113-18)Glocose 3.8 mmol/l P.5-5·51

Questions

1. a) Describe his gas exchange.b) Describe his ilcid-base status.

2 What is the anion gap?

3. Is the acid-base status consistent with methanol ingestion?

.9

Page 94: Arterial Blood Gases Made Easy - I. Hennessey, A. Japp (Churchill Livingstone, 2007) WW

THE ABC IN PRACTlCE

CASE 17

History

A 52-year-old man is being investigated on the urology ward fOT

recurrent renal stones. He also complains of mild fatigue andlethargy. There is no history of gastrointestinal disturbance and he is

not on any regular medi1:ations.

Examination

The patient is weU and clinical examination reveals no abnormalities

As part of the investigations an ABC is obtained.

90

Page 95: Arterial Blood Gases Made Easy - I. Hennessey, A. Japp (Churchill Livingstone, 2007) WW

ArrcrioJ blood go~ On air

23/04/2006U",JI no.: 27634943

10: Roger PorryNonnol

H' 43 nmolIL 13~

pH 737 (735-7..45)

Ito, 4.2 kPo (47-6.0)31.5 mmHg {35-45}

"" 13.2 kf'o I> 1O.6}990 mmHg 1>801

S<o<b 18 mmol/l 122-281

Sf -7 mmol/L 1-2-+21SIb, 99'% I> 98%1

"""'" I 10..4-1.51K 3.0 mmol/t (3.5-51No 137 mmol/l [135-145)

CI 109 mmol/l [95-105)iCo· 1.0 mmol/t (1-1.251Hb 13.0 g/dl 1l1.5-15.5)G1UCOle 4 mmol/l 13.5-551

Questions

1. a) Descnbe his gas exchange.b) Describe his add--base status.

2. CakWate the anion gap_

3. What is the likeliest diagnosis?

THE ABG IN PRACTICE

.,

Page 96: Arterial Blood Gases Made Easy - I. Hennessey, A. Japp (Churchill Livingstone, 2007) WW

THE ABG IN fl:ACTlCE

CASE 18

History

An 18-year-old is admilled to the toxicology unit ha\ing takena large overd06e ofan unknown substarKe 5 hours earlier. Shecomplains of nausea and a high-pitched noise in her ears.

Examination

On examination, she is mildly confused. Her respirations areincreased In both fale <Uld depth. Ex.amination is otherwiseunremarkable.

""~Rllspiratory rOM

Bbxj pl"IIUU'1I

'Tenperatl;,.

0,"

92

100 beals/min26 moottn/min

132/100 mmHg37.6'"<:....

Page 97: Arterial Blood Gases Made Easy - I. Hennessey, A. Japp (Churchill Livingstone, 2007) WW

THE ABG IN PRACTICE

Arteriol blood gos On air10/09/2006Unit no,: 2763.49.43

10: libby Forqh\lor

No<~1

". 38.8 nmoljl (35-45)

pH 7 ..41 (7.35-7..45)

Pto, 3.01 kPo 14.7-6.0)22.6 mmHg 135-45114.1 kPo I> 10.6197.5 mmHg (> 801

Bioo<b 176 mnd/l 122-281BE -8.3 mmolJl l-2-+21,." 99% I> 98%)

!octo.. 1.. (0........ 1.5)

K 3.6 mmol/l 13·5-51No 140 mmol/l (I35-1.451CI 99 mmoI/l f9S-1051ICO· 1.2 mmol/l 11-1.251Hb 13.0 s/dl 111.5-15.5)Gfuco~ 5 mmoI/l 13.5-5..51

Questions

1. a) Describe her gas exchange.b) Describe her add---base status.

2. Cak-ulate the anion gap.

3. What substance is she most likely to have taken?

Page 98: Arterial Blood Gases Made Easy - I. Hennessey, A. Japp (Churchill Livingstone, 2007) WW

THE ABG IN PRACTICE

CASE 19

History

A 24 year old man is brought to hospital by his family after becomingunwell at home. They I't'port thai he has no history of significantmedical illness but has recmtly been undergoing tests for progressivefatigue and weight loss. Over the last few days he has becomeillcrcasingly weak and lethargic and has also complained of musclecramps. They became alarmed today when he appeared drowsy anddisori(,·nted.

Examination

TIle patient appears listless and confused. He has cool peripheriesand pvor C3pillary refill. He is afebrile and lhere is no rash,Iymphadenop<lthy or meningism. Abdominal examination iswlfcmarkable and there are no focal chest or neurological signs.

Pul~ 120 beats/minRespiratory role 25 breot!ls/min8Iood preswre 75/55 mmHg

Temperature 36SC

~ 2_.• -,

.4

Page 99: Arterial Blood Gases Made Easy - I. Hennessey, A. Japp (Churchill Livingstone, 2007) WW

THE A8G IN PRACTICE

Arterial blood gos On_

10/08/2005Unit no.: 45698679310: Fh..r\l~ Wainwright

"",~I

H" 48 nmol/l (35-45(

pH 7.32 (7.35-7.45}

""" 3.) "Po (4.7-6.0)24.8 mmHg (35-451

Po, 13 "Pa I> 10.6)97 mmHg I> 80)

a-b 13.4 IM'OI/l. (22-281

" -1).9ml'l'lo(fl l-2-2JSo, """ (> 98%1

'octo. 3 {O.4-1.5}

K 5.6 mmol/l 13.5-51No 125 mmot/l (135-1451CI 101 rrnmi/l 195-1OS(

""- 1.2 mmotjt (1-1251Hb 13.0 sId!. 11~181

G1uco$e 2.5 mmollt (3.5-5.51

Questions

1. a) Describe his gas exchange.b) Describe his acid-base state?

2. Are there any other abnormalities on th(' ABC?

3. What specific t::reabnent does this patienl require?

.s

Page 100: Arterial Blood Gases Made Easy - I. Hennessey, A. Japp (Churchill Livingstone, 2007) WW

THE A8G IN PRACTICE

CASE 20

History

An 87-year-old man has been found collapsed in the ward toilet.Cardiopulmonary resusdtation was commenced promptly, asno pulse or respiratory effort was detected, and has now been inprogress for 12 minutes. He has a past medical history of ischaemicheart disease. dementia and chronic renal failue.?

Examination

The patient has a Glasgow coma scale score of 3 and appears paleand mottled. The cardiac monitor reveals an agonal rhythm as shownbelow. No pulses are palpable and there is no respiratory effort. He iscurrently being ventilated by a bag and mask on 15 L/min of~.

••

Page 101: Arterial Blood Gases Made Easy - I. Hennessey, A. Japp (Churchill Livingstone, 2007) WW

THE A8G IN PRAGICE

ArJerial blood 90$ On 15 L O2 by bag mask10/02/2004Unit no.: .42333993

ID: [)ovid KingNormal

W 160 nmol/l (35-451

pH 6.' (7.35-7.45)

reo, 4.8 kPa (.4.7-6.0)

36 mmHg (35-451

Po, 32 \;Pa (> 10,6)

240 mmHg (> 80l

Bicarb 3.8 mmol/l (22-281

BE -20 mmol/l 1-2-~2)

So, 100% I> 98%1lackJte 9 {0.4-1.5), 4.5 mmol/l (3.5-5)

No 136 mmol/l 1135-1451

C1 96 mmol/l {95-1 051Co· 1.1 mmol/l 11-1.25)

Hb 14.0g/dl 113.5-18.5)GllICQSe 4 mmoJ/t (3.5-55)

Questions

1. a) Describe his gas exchange.b) Describe his acid-base status.

2. 'vVha1 is his prognosis?

97

Page 102: Arterial Blood Gases Made Easy - I. Hennessey, A. Japp (Churchill Livingstone, 2007) WW

THE ABG IN PRACTICE

CASE 21

History

A 59-year-old male with a history' of alcohol excess presents tothe emergeocy departmc:nt with a 3-day history of 5e\'e"re uppc:rabdominal pain. He now also reels \'ery' breathless. He ddmits todrinking up to 100 units of alcohol per week for the past few wee

Examination

Th{' p.."\tient is in f'vi(jpnt (ji<;tn..,.;.~ and appears very unwell. He istachycardic (120 beats/min) and hypotensive (75/60 mmHg). This marked epigastric tenderness.

A venous blood test taken on admission reveals a grossly elevatedamylase (1890 U/ml) and C-reactJ.ve protein (274 mg/L). A chestX-rayon admission is shown in Figure 34.

••

Page 103: Arterial Blood Gases Made Easy - I. Hennessey, A. Japp (Churchill Livingstone, 2007) WW

Arle1iol blood 90510/08/2005Unil no.: 45679393

10 Dofli~l Corter

THE ABG IN PRACTICE

On 15 L O2 by mask withreservoir bog

H"

pHPro,

Bi<mb

"S""loclo~

•NoCIi(o'

HbGoc~

47 """"It7.33

3.24lPo24.3 mmHg10.8 kPo81 mrnHg14.9 n-wnoI/t-11.8 mmol/l.99%3. 1

3.6 mmol/L1.4\ mmollL96 mmol/l.0.89 mmol/L12.0g/dl16 mmol/L

NOImol

135-"51(7.35-7.AS}

14.7-6·01(35--451I> 10.61I> 80)122-28){-2-+21(> 98%)10.4.-1.5)13.5-5)(135-1451(95-1051(1-1.251p3.5-\8.5)(3.5-5 5)

Questions

1. a} Describe his gas exchange.b) Describe his acid-base St.,tU5.

2. \\'hat is his approximate FiO:z?

3. \Vhat is the most likely diagnosis?

..

Page 104: Arterial Blood Gases Made Easy - I. Hennessey, A. Japp (Churchill Livingstone, 2007) WW

THE ABG IN PRACOCE

CASE 22

Hi5tory

A 35-year-old woman on a g}"naceoJogy ward den~lopsa severe...omiting illness one day after elective steriJisation. She continuesto vomit profusely for a further 3 days. Examination of her fluidbalance chari re\'eals that she is failing to kc:..oep up with her fluidlosses but has not been prescribed intra"enous fluids.

Exomination

The patient appearsdehydraled WIth reduced skin turgor and drymUCO\lS membranes.

Abdominal examination is unremarkable.

p",~

Re~pirotory rale

Blood preswre, lemperOTure

I0,"

100

100 beab/m,n10 brectfu/mirl160/100 mmHg36_6~C.."

Page 105: Arterial Blood Gases Made Easy - I. Hennessey, A. Japp (Churchill Livingstone, 2007) WW

THE ABG IN PRACTICE

Arterial blood gas On air23/08/2006Unill1o.: 27634943

ID: Jenny A1egranzaNormal

H' 36 nmol/l 135--451

pH 7.44 17.35-7.45}

/to, 6.4 kPo 14.7--6.0)

48 mmHg 135-4511'>, 11.1 kPo I> 10.6)

83 mmHg I> BO)Bicarb 32 mmol/l 122-281BE +4 mmol/l l-2-+2}

51'>, 96% I> 98%1lactate I 10..4--1.5)

K 3.0 mmal/l p.5-5}

No 133 mmol/l 1135-1451

(I 91 mmol/l 195-105)

iCo' 1.0 mmoJ/l 11-1.25)

Hb 11.°9/dl 111.5-15.5)

Glocose 5 mmol/l 13.5-5.5)

Questions

1. a) Describe her gas exchange.b) Describe her acid-base status.

2. \Vhat electrolyte abnormalities are present?

3. \Vhat lreatment will correct the acid-base <lbnorm..ijty?

101

Page 106: Arterial Blood Gases Made Easy - I. Hennessey, A. Japp (Churchill Livingstone, 2007) WW

THE ABC IN PRACTICE

CASE 23

History

An 8-w('('k-old child is brought to the emergency department, withweight loS/> and projectile vomiting. The paIT'nts report that he hadan uncomplicated delivery with no postpartum complications.He initially fed well, appeared to be thriving and gave no causefor concern but has deteriorated markedly over the past 2 weeks,vomiting all of his meals and now losing weight.

Examination

The child is agitated, crying and is malnourished. His mucousmembranes are dry and oncxamination of his abdomen a small mis found in the epigastrium.

102

Page 107: Arterial Blood Gases Made Easy - I. Hennessey, A. Japp (Churchill Livingstone, 2007) WW

THE ABG IN PRACTICE

capillary blood g05 On air

05/10/2005U"iIIlO.: 28734943ID: Richard Barter

Normol

W 29 Ilmol/l {35-451

pH 7.54 17.35-7..451

Pw, 6.1 1&'0 14.7-6.0}

.45.8 mmHg tJ5-45111.2 ~Po (> 10.6)

80 mmHg (> BOlBiUlrb 37.5 mmo!/l (22-2BI

BE +14 mmol/l (-2-+21.

"'" 99% (> 98%1

laerate 1 (0.4-1.5)

K 2.5 mmol/l 13·5-51No 135 mmol/l (135-1451(I 86 mmo!/l (95-105)

iCo' 1 mmol/l (1-1.251

Hb 1B,O gJdl (13.5-18.5)

Quesfions

1. a) Describe his gas exchange.b) Describe his acid-basc status.

2. Considering the pH and He03: is the PaC~ IUgher or lowerthan you would expect?

3. What is the underlying diagnosis?

'03

Page 108: Arterial Blood Gases Made Easy - I. Hennessey, A. Japp (Churchill Livingstone, 2007) WW

THE ABG IN PRACTICE

CASE 24

History

A 36-year-old pregnant woman 011 the maternity ward complainsfeeling short of breath. She ha~ no other symptoms and no relevantpast medical history.

Examination

On examination, the patient is heavily pregnant but appearsotherwise well. Examination of her chest re....eals no abnormalities.

PubeRespiratory role

Blood prenure

Temperature

0,%

110 beots/mirl

20 breaths/min112/100 mmHg36.6"(:

99%

An AI3G is performed.

104

Page 109: Arterial Blood Gases Made Easy - I. Hennessey, A. Japp (Churchill Livingstone, 2007) WW

THE ABG IN P~ACTICE

AfMriol b600d gas On air18/08/2005l,klill'lO.: 27634943

10 julie DcnoldsonNeomal

H" 35 nmol,/l (3)-<5)

pH 7.45 (7.35-7.45)

Ito, 4.9 kPo 14.7-6.0135 mmHg (35-.451

Po, 4.7 Id'a I> 10.6)

35 mmHg 135-t5)Bicarb 24.0 mmol/L 12WOl

" 2 mmoll\. 1-2-...2)

SIb, 74% I> 98%l_Ie I 10·4-1.51, 3.6 mmol/l 13.5-5)

No 1381nf1'1oOl/l. (135-1451C; 1Q.4 mmol/l. (95-105JiCa' I. U mmol/1 1I-1.2S)tf> 13.0 g/dl (11.5-15.5j

GI.<o~ 5 mmol/l (3.5-5.5)

Questions

1. a) Describe her gas exchange.b) Describe her itdd-base status.

2......'hat is the mosllikely ell.ptanation lor the low P~?

'0'

Page 110: Arterial Blood Gases Made Easy - I. Hennessey, A. Japp (Churchill Livingstone, 2007) WW

THE ABC IN PRACTICE

CASE 25

History

A 55-year-old woman on till> orthopaedic ward complains ofsuddcn-onset breathlessness and pain on the left-hand. side of herchest. She underwent elccti\·e knee replacement surgery 4 dayspreviously and has!x.'('3\ immobile in bed since the operation. She isotherwise well with no rele ...·ant past medical history.

Examination

The patient appears well but slightly short of breath. Other thanmild tachycardia and tachypnoea, examination of the cardiovasculand respiratory systems yields no positi ....e findings and there is noclinical evidence of deep-vein thrombosis.

A chest X-ray reveals no abnormalities and an electrocardiogram

shows only sinus tachycardia

""~Re1piratofy r<R

8Iood .......Tem~olure

0,%

'06

98 beets/"'in20 l:lI'eoh/llIifl160/IOOrnmHg

36.6'"<:99%

Page 111: Arterial Blood Gases Made Easy - I. Hennessey, A. Japp (Churchill Livingstone, 2007) WW

",terial blood go~ On air

10/08/2006Unit no.: 27634943ID: jill Arcl1erson

Normol

W 36 nmol/l (35-451

pH 7.43 (7.35-7.45)

Pco, A,9!o:Po (4.7-0.0)

37 mmHg 135-451fb, 12.1 kPo (> 10.61

91 mmHg (> 80)

Bicorb 25.8 mmol!l (22-28}

BE -1.8 mmol/l (-2-+2)

SPC, 99% (> 98%}

lactate 1 (0.4-1.5)

K 3.8 mmol/l (3.5-5)

No 136 mmol/l {135-1A5)(1 99 mmol/l 195-105)

iCo' 1.2 mmol/L 11-1.251

Hb 10.09/dL 111.5-15.5)

GllJco~e 5 mmol/L (3.5-5.5)

Questions

1. a) Describe her gas exchange.b) Describt'o her acid-base status.

2. What is the A-a gradient?

J. Does she require <my further investigation?

THE ABG IN PRACTICE

'07

Page 112: Arterial Blood Gases Made Easy - I. Hennessey, A. Japp (Churchill Livingstone, 2007) WW

ANSWERS

CASE 1

1. 0) Type I respirotory impoinMIIt {ll'lOlhorotelbl Uncompensated ,e$plrotory aftlC:~osis

2. Ye53. Ves

Ihis patient has modE't'atc Iype 1 respiratory impairmf"l"ltHyperv~hlation is an appropriate response to~ hypoxacmiaand seno;ation of dyspnoea and has resulted in a. mild alkalaemia(remember lhut metabolic compensation does not occur in rESponseto QCute n..~piratory acid--base disturbance).

1nc lUJT@('t manal';cment for his rondition is supplt>lJlelltal oxyg~to COtTfft the hypoxaemia and appruprialP antibiotic; to ~at theinfection.

In a patient sIKh as tlus, with moderate hypoxaemia and novt'fltilalory impairment, monitoring by pulse oximetry is moreappropriolte than n>peated ABG sampling. lru.lications for furtherABC analysis would include f'igns of exhaustion or hypen:apnia(p. 23) or a further :.igmficant~ in~

109

Page 113: Arterial Blood Gases Made Easy - I. Hennessey, A. Japp (Churchill Livingstone, 2007) WW

ANSWE~S

CASE 2

1. 01 QfOflic I)'pe 2 respirolofy impoirmenl

b) Compell$Oled f~irolofy ocidosh

2. Chrooic type 2 respirorory irrpoirment due 10 morbid obesity

At first glance it may be difficult to determine whether this ABCrcp~ts respiratory acidosis with metabolic compensation ormetabolic alkalosis with respiratory compensation, as both givea high HC03 and high PtiCOz. The best due is the pH (or H')which, ahhough just within the normal range, is on the brink ofacidaemia. ThIS would represent overcompensation for an alkalosisand. therefore, sugg~ an acidosis as the primary abnormality(overcompensation does not occur). The mildJ}' impairedo)()'genation is consistent with the degn.-e of h}l>o'H,:ntilation.

1be likeliest caU5e of chronic type 2 respiratory impairment in thiscase is se\"ere obesity. Around 20% of individuals with a body massindex greater than 40 have chronic hypercapnia from restrictedventilation (pickwickian syndrome).

110

Page 114: Arterial Blood Gases Made Easy - I. Hennessey, A. Japp (Churchill Livingstone, 2007) WW

ANSWERS

CASE 3

1. 01 Mild Iype I fespirotory impairment (wirfl morhd hyperventilation)

hi Uncompensated resplfclory olkalosis

2 Putmonory embolism

This poltient is a young. fit. non-smoker with no history of lungproblems but, despite hyperventilating (low PacOV. has a P~ belowthe normal range. indicating impaired oxygenn.tion. Given the recentlong-haul flight and absence of clinical and X-ray abnormalities. themost likely cause of her breathlessness and impaired oxygenation ispulmonary embolism and she must be investigated accordingly.

This is oneaf the clinical situations wheTe ukulation of the.\-a gradient can be helpful (more so when the P~ is just .....'ithinthe normal range). As shown below, it is signiflcantly elevated,indicating the presence of VIQ mismatch.

IA-<J grodient - PA~ - Po~PAD2 _ (0.2/ l< 93.81- (3.9 l< 1.2)

_19.7_4.7_ 15/t;Po

A-o grodienl_ 15 - 10 3

.4.7 ;aorm <2.6 kPo)OR

PA02_{O.21 l<713J-/29.3x 12)

- 150- 35_ 115

A-o gfodieot • 115 - n• 38 mmHg (norm <20 mmHg)

---------'----'------'

,..

Page 115: Arterial Blood Gases Made Easy - I. Hennessey, A. Japp (Churchill Livingstone, 2007) WW

ANSWERS

CASE 4

I. aJ Acute type 2 re1plrafory fodureb] Uncompensoled rllspirotOf)' ocidosis

2 Opioid toxicity

3. Op,otd ontogonisl

Opioids haw a depressant effecl upon respiration and may lead10 acute \;t'I1tilatory failure (type 2 respiratory failure). This elderlyman has received a large amount of morprune in a short period oftime and exhibits pinpoint pupils. malting opiok! toxicity by farthe likeliest cause of his severe ventihllory failure. Since metabolicrompcnsation takes several days to occur, the acute respiratoryacidosis has produced a SC"t'Il! acidaelJlia.

HisP~ though within the normal range, is lower than expectedf~ a pilllt!'ni breathing 28~o Oz, and more or less consL'itent with thisdegree of hypoventilation.

In addition to basic life support meilsures, he should beadminislered an opioid ant'1.gonist (e.g. naloxone), to re\'erse therespiratory dcp~ion, then closely monitocro 10 ensure sustainedimpro~·em~nt.

112

Page 116: Arterial Blood Gases Made Easy - I. Hennessey, A. Japp (Churchill Livingstone, 2007) WW

ANSWERS

CASE 5

, :::I) Type I re$pira:OfY Imparmen1lmoderalel

bl Normal a<;i~tl(He balooce

2 Yes

l1'e above ABC IS helpfulln guiding the correct t~atment for thispol'cnt with acu~ exacerbation of chronic obstructive pulmoRdrydl~~;a5€'. Patients wilh this condition are often prescribed inadequateox\gen for feM of suppressing hypoxic drive (and thcJ:ebydepressing ventilation), but this is onJy an issue in patients with..-'IM'I!.:: /yF1I/' 1 respiratory failure (indicated by t PlICO:!. and t HCD,).Thl., patll~nt has typr 1 resptratory impairment and will not rely onh) p'lxic drive.

Although this patient is likely to ha ....e a chronically low 1'>1Q:, the3(1.)t", deterioration in his symptoms and exercise tolerance suggestsa rurther n.~tdecline from his nonnal baselinf-. lmportanlly, l"·ena small drop in P~ around this }('vel (steep part ofOr-Hb curve)may C<lU5e a marked reduction in Sa<l.!, compromising D,; deliveryto tissues. Thus O2 is required both to alleviate symptoms andto pre....ent the development of tissue hypoxia, and sJwuld not b<­withheld for fear of precipitating hypoventilatlon.

Page 117: Arterial Blood Gases Made Easy - I. Hennessey, A. Japp (Churchill Livingstone, 2007) WW

ANSWERS

CASE 6

1. 01 Acute Iype 2 respirotory impoirmet'll

bI Uncompenwted respirokJfy ClC~1

2. Not

This patient with an acute exacerbation 01 chronic obstructivepuJrnonary disease has been struggling to overcome severeob6truction to airflow over a period of hours to da~"S and is nowexhausted from the increased work of breathing. As a result, hisalveolar ventilation is declining. leading to acute type 2 respiratoryfailure. Th..is JTh1.y complicate type 1 respiratory failure from anycause. not just from chrunic obstructive pulmonary disease. Therisingp~ is. therefore, 'lOt due to diminished hypoxic drive andremoving his oxygen willl10t correct it. Indeed his F~ shouldprobably be incT('~ (in addition to other treatment) as he remaisignificantly hypoxaemic.

Remember" thai. with acute respiratory acidos.is, there is no timefor metabolic compensation to develop and a dangerous acidaemi.l.de"'elops rapidly. Adequate \"{~ntilationmust be restored, as amatter of urgency. to COrrecl the PIlCOz. Possible measures. in thiscase, include a respiratory stimulant (e.g. doxapram) or, preferably.non-invasive ventilation. If these fail, mtuhalion and mechanicalventilation may be required, If considered appropriate.

".

Page 118: Arterial Blood Gases Made Easy - I. Hennessey, A. Japp (Churchill Livingstone, 2007) WW

ANSWERS

CASE 7

01 Type 1 r~i,oklry impoirmef'll

bJ M Id re~rotory ollolosis bo onced by mild me'Oboi c ocidc~'s

p,kely two p"mory processesl2 Aspiration pnevmo'l 0

J Se¥efe

nu.. patient has mild to moderate hypoxaemiJ despite recei\'ing ahIgh Fi02and therefore has severe impairment of oxygenation The,;hf;htly low PilCo.: indicates that ventilation i" adt'quate, so this l~

II'PC I n..~piratory impairment. The probable expl,mation for the mildIllct3boli.::: acidosis is lactic acidosis resulting from tissue hypoxi",

The lustoI)~ examination findings and chest X.ray all suggcst aJi.lgOO5is of aspiration pneumonia.

nus patient is se\"erely unwell and any further decline inher P0O:: could be catastropttic (on the steep part of the OrHb.aturation curve). Her 0:l therapy should be adjusted a.., necessary10 mamtain ht.-r 5Doz abo\"c~.. and she should be managedon iI htgh--d.epcndency unit with dose monitoring for- signs ofdeterioration.

'"

Page 119: Arterial Blood Gases Made Easy - I. Hennessey, A. Japp (Churchill Livingstone, 2007) WW

ANSWERS

CA5£ 8

1. '" Chronic type 2 re~rotocy irnpaiflMnt

bt Compensated respirolory ocidosis2. PO,3. PC~ and HC03

Acute exacerbation of chronic obstructive pulmonary disease is acommon medlcal emergency and this case illustratt's key principles.

], The fp~ shows the patient has type 2 respiratoryimpairment (ventilatory impairment).

2. The f HeOl tells u.s it is chronic type 2 impairment (sincemetabolic compensation I~ time to develop),

3 Even in chrt'lnic type 2 impairment, a further DiCute rise inPoco: would lead to an acidaemia. The pH hen? is normal, soIhe PtKOz h.1S not changed appreciably in the last few days(i.e. there is no acute-on-<hronic rise),

4. His PtK>:z.lS likely to ha\'e dropped, leading to the increasedbreathlessness and marked decline in eJt"erc1se capacity: bela PIJ~ of 8 kPa (60 mmHg) ev<.-'ll small falls may cause a rna'decline in $QO;z (steep part of the cun'e).

5, As this patient has chrortic hypercapnia, he may rely onhypoxic drive as a stimulus to ventilatlon,

6. The goal is to ensure adequate oxygL'Oation (we must notignore his hypoxaemia) without depressing ventilatory drive

"6

Page 120: Arterial Blood Gases Made Easy - I. Hennessey, A. Japp (Churchill Livingstone, 2007) WW

ANSWERS

CASE 9

1. 01 AcullH)l'Khronic type 2 re5pirotory imfXIirm6rltb) Partially compensated respiratory acidosis

2 Excessive supplemental O2

Care must~ taken when prescribing supplemental Oz to patientswith chronic type 2 respiratory failure. The aim is to reverse,my recent worsening ofhypoxaemia and allow adequate tissueoxygenation, without depressing ventilatory drive through anexcessive rise in PaO:l.

Most authorities recommend controlled O2 therapy using a fixed­concentration mask at an initial concentration of 24-28%. Theresponse to therapy must be dosely monitored with frequentclinical assessment and repeated ABC measurement. Note thatpulse oximetry is /lot an adequate substitute for ABC in thesecircumstances as knowledge of the Sao:z alone does not permitassessment of ventilatory adequacy.

The latter point is well illustrated in this case as the-patient now h,lSlife-threatening acute-on-ehronic respiratory failure despite a normal~. The rising Pac~ must be rapidly checked and reversed throughimproving ventilation. Potential strategies include reducing inspired0, cnncentration, respiratory stimulants or assisted ventilation.

117

Page 121: Arterial Blood Gases Made Easy - I. Hennessey, A. Japp (Churchill Livingstone, 2007) WW

ANSWERS

CASE 10

a) Miki type I re$pimtary impairment

hi Normal2. TheP~ (high end aI ncl'TIal ronge)

J. Life.lhreatening ahad

This patient has sen~ral features of a ~"ere asthma attack but itis the high-normal PilCO} that is the most worrying aspect of herpresentation and makes it a life-threatening attack. Patients withacute ex.ocerbalions of asthma should have a low P~ on accountof the increased respiratory rate and effort (talveolar \·entilation).A level)o 5 kPa (37.5 mmHg) suggests the patient is strugghng toovercome the obstruction to airflow and. perhaps. beginning to tirefrom the effort of breathing. Consequently. her Paco:: signals alife-threatening attack.

The intensh·e care unit should be informed irnmediaklr of anypatient with acute se,·ere asthma and Iik--threatening features.P<1tients must receh"e intensive tre<1tment and monitoring, includingrepeated ABC me,lsurcmomls to assess n.--sponse and identiIy thenCl'od for intubation.

n.

Page 122: Arterial Blood Gases Made Easy - I. Hennessey, A. Japp (Churchill Livingstone, 2007) WW

ANSWERS

CASE 11

I 01 Hyper\/'llfllilorioo lp"morylbl U~peruoted respiratory oll:alosis

2. Low ionized coki\,lll'l3 Psychogenic hypervenlilotioo

This is a classic clinical picture of psychogenic hyperventilation.The ABC is entirely in keeping with this diagnosis as it revealse\-idence of hyperventilation (low PIl'O:>V with normaJ oxygenation(nannal pQO:z) and a normal A-a gradient.

Note that the HC~ (and BEl is norma.l as there has been insufficienthme for metabolic compensation to OCXUf. Consequently thert'duction in FiKOz has caused a marked alkalaemia.

Anothet" point to noll' is that the concentJation of ionized calciumm the blood is afft>eled by the pH of the specimen. since H' ionscompete with cakium for binding sites on albumin and otherproteins. Therefore, as the number of H+ ions drops (alkalaemia),more caldum is bound to albumin, causing serum Ionized calciumlevels to fall. This is the most likely cause of the numbness andtingling.

Care must be taken in ruling out other cardiovascular andrespiratory pathologies before ascribing s)mptoms of chest pain andshortness of breath to psychogenic hyperventilation.

119

Page 123: Arterial Blood Gases Made Easy - I. Hennessey, A. Japp (Churchill Livingstone, 2007) WW

ANSWERS

CASE 12

I. 0) Normol- bul w.ere hypoxaelftiobl COlTIpeoSOled metobolic ocidosil

2. COfbon monoxide poisoning

J. Soc,

Carbon monoxide (CO) poisoning commonly presents with nausea,vomiting. headache and ronfU5ion. CO saturations correlate poorlywith symptoms but levels above 5O'r~ may cause cardiac arrest andseiz.ures.

Why is the patient hypoxaemic?

CO binds to hacmogJobin (Hb) with 200 timts the affinity of~ so iscarried on the Hb molecule (as carboxyhaemoglobin) in preference to~. As a consequence. the perrentagc of Hb saturated with Oz - the~ - is maTIcOOI~' redl.K."'ed in CO poisoning. f",'cn when the p~is very high. Since the overall Oz content of blood is determined bythe Sooz and Hb concentration,~ dclivcry to tissues is inadequate(tissue hypoxia), le."lding to lactic acidOl'iis.

Why does .5cJo, appear to be normal?

Most pulse oximeter.> are unable to distinguish carboxyhac:moglobinfrom oxyhaemoglobin, so fail to reflect the truc~ in CO poisoningIn ABC analysis. the~ is not norrnaUy measured but simplyCOlIeu1atro from the measured PaOz. The Jailer parametcr is based onlyon free, unlxlUnd O2 molccules SO is unaffected b}' the presence ofCO.

120

Page 124: Arterial Blood Gases Made Easy - I. Hennessey, A. Japp (Churchill Livingstone, 2007) WW

ANSVVERS

CAS! 13

I. 01 HyperverJtilation _ 00 impairment of oxygenol;Of\ bul hypoxoemio

se<:ondary 10 onoemio (P.I 81bl Uncompen~ted re$pirotory alkalosis

2. Anoenio (Hoemoglobin 6.813. Reslolft hoemog!obin {Hbt <:onc:anllo!ion (I.g. blood transfusion, ilon

replocementl

This patil.'I1t appears to have severe anaemia, most likely due to irond"fKicrlC)' from dmmic ra"tal bll"l'dlng. This should be rontirmedwith a formallabocatory sample ('full blood counr).

There is no impairml...'I1t of ventilation or O2 transfer so Pae>. andSaO:! are normaL However the vast majority of O:l: in blood is carriedby Hb, so the overall O:l: conlmt of her blood is low.

Hyperventilation is a nonnal response to the sensation ofbreathlessness and increases the PaO:l: slightly" How€\u, in theabove context, this has very little impact on blood O:! content as theavailable Hb molecules are already fully So'lturated:

For the same reason supplemetal O:! would also fajl to improve O~content significantly. Indeed the only effecli\"e stTatt:.-gy is to increaseHb concentration. This could be achieved rapidly. by a bloodtransfusion or more gradually by iron replacement.

.2.

Page 125: Arterial Blood Gases Made Easy - I. Hennessey, A. Japp (Churchill Livingstone, 2007) WW

ANSWERS

CASE 14

oj Hypen!entilolion (secondary!bj Partially compenwled meklbotJe acidosis

2. Mesenteric ischaemia

What does the ABC tell us here? There is acidaemia due loa severemetabolic acidosis and the elevaled lactate tells us it is a loctlcacidosis. Lactic acid is produced by tissues recch'ing an insufficientsupply of O2 bul oxygenation is normal (note the PtlOJ is appropriatefor an F~ of -40%) and there are no clinical signs of shock (e.g.hypotension. cold peripheries) suggesting there is no generalisedproblem of~ delivery to tissues.

In fact, the source of lactic acid here is bowel. The patient hasmesenteric ischaemia, in which blood suppl}' to the bowel wall is

impaired due 10 occlusion of an artery by a thrombus or embolus.In the absenct" of an adequate blood suppl)~ bowel tissue becomeshypoxic and must rely on anaerobic mrlabolism (producing lactateas a byproduct).

Mesenteric ischaemia is a difficult diagnosis to make. as presentingsymptoms, signs and routine im'estigations are all non-specific.diagnosis should be considered in patients with minimal abdOUll1\l1examination findings despite severe pain. especially in the presenceof a lactic acidosis.

122

Page 126: Arterial Blood Gases Made Easy - I. Hennessey, A. Japp (Churchill Livingstone, 2007) WW

ANSWERS

CASE 15

oj Hyperventilation (lecondory)

bl Se-e rretobolic ocickW$ with partial compenKition

2. AIltcn gop • 0.4 1 + 4.6) - (96 + 6) • 43.61nonnal. 10-12)":: Diaberic ketoacidosis

In dmbetic keto.1cidosis, severe insulin deficiency leads tohrpcrglycaemia and incrt'ased metabolism of fats. 'The bl't"akdown(If fats produces ketone bodlCS - smaU organic acids - which providelin alternati'l;e source of energy but can accumulate, leading to arrofound metabolic acidosis. [t is the ketone bodies thai account forI~ raised anion gap.

In this case the acidosis has overwhelmed not only the kidneys'ability to excrete an acid load but also respiratory compensatorymechanisms. 1herl' is therefore a severe and dangerous acidaemiadespite near-maximal respiratory compensation.

An equally important problem in diabetic ketoacidosis is theprofound osmotic diuresis resulting from hyperglycaemia that leadsto severe dehydration and electrol}'te loss.

Page 127: Arterial Blood Gases Made Easy - I. Hennessey, A. Japp (Churchill Livingstone, 2007) WW

ANSWERS

CASE 16

1. 01 Hyperventilation I~andary'

bJ Sevel8 metabolic ocidalil wi'" portial compenWlioo2. 323. Yel

Methanol ingestion can be fatal in doses as small as 30 mL Methanolis metabolised by the liver to produce formaldehyde and fonnicacid. Accumulation of formic add leads to a profoWld acidosUi witha raised anion gdp. It alsoca~ ocular toxicit)' and may result inpermanent blindness.

Anion gap.: (Na /1361 + K 14.51)- (CII991 +H~ 19.51)"" 140.5-108.5'" 32

Treatment for methanol poisoning is complex but often involves theadminislTation of ethanol, which inhibits the conversion of mcth..molto its more toxic metabolites.

12.

Page 128: Arterial Blood Gases Made Easy - I. Hennessey, A. Japp (Churchill Livingstone, 2007) WW

ANSVVERS

CASE 17

0) Slighl hypenoenllotion lsecondory)

bI Compersated metabolic ocidosis

2 Aniongop -1137+3.0:1-(109+ lSI_ 138-125

• 13 (normoq3 RenolllJoolor ocidcsiJ (type II

The diffurentiaJ diagnosis of normal anion gap acidosis is rcL"ltivelyn.\rrow and, in the absence of diarrhoeal s}·mptom.<;, ~al tubulari1cdosis (RTA) is the likely cause. The history of renalstoncs anda...."ocialcd hypokalaemia also supports the diagnosis_

In type 1 RTA,lhe kidneys fail to to secrete H' ions into the unne inl''(\--h,mge for Na' ions. This leads to e>.:cessh·c loss of HCO:J in theurule, resultitlg in an acidOSIS. To maintain e1cclroneutrality, e>.:uaCI U.lnS are n..-taincd (so it is a "~rchJornt!mjc acidosis). Becau~0- isa m{'asured ralher than unmeasured anion, lhere IS no increase in theanton gap.

Type 1 RTA is often complicated by renal stones as calcium lends torrecipilate in alk."lline uriTll', fonning stones.

H}'pokalaemia results because "Ia· ions are exchanged for K· ionsIlt>l...ad of H· IOns.

.25

Page 129: Arterial Blood Gases Made Easy - I. Hennessey, A. Japp (Churchill Livingstone, 2007) WW

ANSWERS

CASf 18

1 01 Hypervenlilofionb] Compensated metabolic ocu:I01.i1. 0( metaboLe ocid01.is with

concomitcmt re1.pFrolory oIkoklsis2. 27J. Aspirin

5.llic~'late poisoning may c.'luse both a pnmaI)' respiratory alkalosis(direct stimulation of the respirator), centre) and a primary metabaracidosis as saliC}'l.1te is an acid (it may also promote lactic acidformation). It is therefore not pos..o;;ible to confidently ascertainwhether or not the hyperventilation is due to a primary cffe<:t of theaspirin or is a response to the metabolic acidosiS.

The diagnosis would be confinned in this instance by takingsalicylatlt bels.

n.

Page 130: Arterial Blood Gases Made Easy - I. Hennessey, A. Japp (Churchill Livingstone, 2007) WW

ANSWERS

CASE 19

1. oj HyperventilatIon lsecondary}hI Severe metabolic oddosi~ with portiol COO'IpenlOtion

2 Hypo"IQIfOOMIO II No}. hyperl.oloemio I t Kj, hypoglycoemio

I • glucose), ro sed b:lale

J. ItllraYeOQUs l:Oflicoslero:ds

This patient is likely to have adrenal insuffidency, a condition inwhich the adrenal glands fail to produce sufficient amounts of thehormone cortisol (and, in some cases, aldosterone). It often presentsnon-specifically with fatigue. malaise, anorexia and weight loss canbe easily overlooked or misdiagnosed.

Llck of adrenal honnones causes sail and water depletion and loss ofvascular tone which, as in this case. may lead to dramatic circulatorycollapse (acute adrenal crisis). The circulator), shock is the cause ofthe severe lactic aadosis.

Patients also ha\'t" an inability to mobilise glucose ~r\res,h<'nce thehypoglycaemia, and characteristic electrolyte abnormalities(j Na: I K).

In addition to basic lire suppoc-t and fluid resuscitation. the mainintervention likely to improve this patient's condition is theadministration of intravenous hydrocortisone.

127

Page 131: Arterial Blood Gases Made Easy - I. Hennessey, A. Japp (Churchill Livingstone, 2007) WW

ANSWERS

CASE 20

1. 01 Normal (i .•. soliJocIory Yllflhlatioo and oxygenotion being

ochieyed by bog orKl roosk)

~ $eYere metabolic acidosis funcOlflJl8NOtedl2. Prognosis Very poor

1n i\ cardiac arrest case, the ABC has several uses. It allowsone 10 dett'mline the adequacy of ventilation (iII this caseprovided manually by bag and mask), to identify the presence ofhyperic:alaemia (o~ of the reversible causes of cardiac arrest) andmay providE.' valuable progn~ticinfonnatioo.

Hen', despite the evident success of bag and mask \'entilation inelinunating CO} and oxygenating blood, the patienl has a profoundlactic acidosis secondary to over,o..-helming tissue hypoxia. Althaugtills may be multifactorial (e.g. the umkrlying disease process;hypoxacmia prior 10 bag and mask \'cntilation), the most importancall.<;£' is madequale: tissue pet'fusion due 10 loss of cardiac fWlction.

This polticnt has a grave prognosis. 11le level of acidosi5 is unlikelyto be compatible with life and, given his unfamurable rhythm(an <lgonal rhythm does not respond to DC cardioversion andoften signifies a 'd~ing heart'), age and ro-morbidili~,successfulr('Su:>olation is extremely unlikely.

128

Page 132: Arterial Blood Gases Made Easy - I. Hennessey, A. Japp (Churchill Livingstone, 2007) WW

ANSWEIlS

CASE 21

0) Type I relpiratory impairmenl 1~V'8fel

bl Severe fl'Ieklbolic acidosil with partial compen$Olion2. 0.6-0.8 f- p. 1613 AcIOO par'lCfealllil

The diagnosis of acute pancreatitis is based on the history. examfindings and increased serum amylase: the main use of the ABC is in:Jelping to assess illness severity.

The severe lactic acidosis (high anion gap and incremled lactate)indicates m.uked tissue h)'J'Oxia. The main caUS(' is impaired bloodsupply to bssuesdue 10 circulatory collapse. This occurs as part ofthe systemic inflammatory response in pancreatitis and must beurgently corrected with aggressive fluid resuscitation:t. vasopressoragents.

A POO:1 of 10.8 with an FiO:z of 0.6--0.8 implies severe respiratoryimp.linnent, and together with chest X-ray appeara.nces, suggeststhe devclopment of acute respiratory distress syndrome (anInflammatory lung condition). Although PaC>]. is currently adcquate,1m:. p.ltient may require ventilatory support if he deteriorates further('r tires from the increased work of breathing.

Finally, the ABG also shows a high glucose and low calciumconcentration - both of which are adverse prognostic fadors in acutepancreatitis.

Th£ patient should be transferred immediately to a critical care('TWlronment for intensive moniloring and supportive treatment.

129

Page 133: Arterial Blood Gases Made Easy - I. Hennessey, A. Japp (Churchill Livingstone, 2007) WW

ANSWERS

CAS022

I. 0] Mild type 2 re~1ofy impairment f<:ompen$OlOr)' rEt$ponse}

b) Compon$llled metabolic alkalosis2. ElKtrolytes Hyp::*alaemio UIQ

Hyponotroemio UNolHypochb..-n.o ucq

3. Treatment Auid and electrolyte repboement

Vomiting causes loss of H+ ions in gastric juice. The normal responseof the kidneys to loss of H+ ions is increased excretion of HCO,- torestore acid-base ba1aoc'e. So why dots this not happen?

The reason is that persistent \'orruting also leads to fluid, Na, CI and

K depletion. In these circumstances the o~'erridinggoal of kidneys is

salt and water retention.

Under the influence of a honnone called aldosterone, Na+ ions

are retained at the expense of either K+ or H+ ions. Lf K+ ions wereplentiful, H+ loss could be minimised, but K is also in short supply soboth are lost (worsening both the alkalosis and hypokalaemia).

0- depletion also limits HCO)- excretion as there must be enoughnegatively charged ions in blood to balance the positi\'e!y chargedions (eJe:tlOneutrality).

Thus, in this case, intravenous replarement of fluid and electrolytes(Na. CI, K) would allow the kidneys to excrete more HCOJ;ooile<:ling the alkalosis.

•30

Page 134: Arterial Blood Gases Made Easy - I. Hennessey, A. Japp (Churchill Livingstone, 2007) WW

ANSWfllS

CASE 23

I , 0) Mild type 2 respiratory impoirmtmllcomper'lsolory responsefb) Nlelabolic alkalosis with partial compensation

2. lower

3, Pyloric stenosi~

Congenital pyloric stenosis is due to hypertrophy of the gastricoutflow tract ~n the first 6 weeks of life. This obstructs flow betweenstomach and duodenum, leading to projectile vomiting and inabilityto absorb nutrients.

As in the preceding cast', persistE!l1t vomiting has caused significantloss of H" ions, triggering a metabolic alkalosis. Again this lS

maintained because the associated fluid, Na, a and K depletionpre\'cnts the kidneys from increasing HCOJ excretion. However, inthis case there is a severe alkalaemia for two reasons.

Firstly the metabolic alk.1.losis is more severe. This is mainly dueto the greater duration of vomiting but also becau;;e obstructionbetween stomach and duodenum prevents HCOJ f~om theduodenum being lost in vomit.

Secondly, there is minimall'E'Spiralory compensation. Given Ihesevere alkalaemia, one would predict a greater rise in PacC'1. lhisis probably explained by the child's distress, which provides anadditional respiratory stimulus blunting the compensatory response(Le. a mild primary respiratory alkalosis).

.31

Page 135: Arterial Blood Gases Made Easy - I. Hennessey, A. Japp (Churchill Livingstone, 2007) WW

ANSWERS

CASE 24

1. 01 Appearance of .evere type I unpiratory impairment

bl Normal acid.bose stalus2 Explaootion Yer100s sample

1be ABC result suggests severr. Life-threatening hypoxaemia butthe patient appears dinicaUy well with onl)" mild symptoms and nosigns of ma;or respiratory compromise. Moreover. there is a markeddiscrepancy betWl.'erl the~ as measured by pulse oximetry (m,,)and thai calculated on the ABC (74%). By far the likeliest explanation.is that the sample was obtamed from a vern rather than an artery. A!Cpt-oat ABC should be performNl.

It IS important to ensure that an ABG sample is obtained from anartery before using it to assess the P/lCIo;.. In addition to the pointsabove. non-pulsatile flow and the need to draw back on the syringeat the time of sampling sugge'!it venous blood.

132

Page 136: Arterial Blood Gases Made Easy - I. Hennessey, A. Japp (Churchill Livingstone, 2007) WW

ANSWERS

CASE 25

I. al NOfl'l'ol gas exchongeb) No'mo1 ocid-bose stotus

2. A-c gradient 19lPa!1S mmHg lnaflno~

3. Ye$(mu!>I edude pul"onory embolismj

This case is included to demonstrate the limitation of ABGs and theunportance of considering the clinical context, The ABC is entirelynormal but the patient is at high risk of pulmonary embolism,given her ret:ent lower-limb orthopaedic surgery and subsequentimmobility. Moreover, she has now developed sudd61.-onsetpleuritic pain, breathlessness and tachycardia, unexplained by initialInvestigations.

While the finding of imp<lired oxygenation would have lent weightto a diagnosis of pulmonary embolism, a normal ABC resultnever- excludes it. She therefore requill$ appropriate imaging (e.g.I:entilation-perfusion scan or computro tomography pulmonaryangiogram) to rule oul this diagnosis.

A-c gradient. PA0:2 - Pro,.

lfO 21.9381-14.9 x 1.2)}- 12.113.9 - 12.1.. 1.8 U'a lnormal.< 2.6 U'aj

'«1[0.21 x713J-lJ7x 1.2}j-91106 - 91

! .. 15 mmHg (normal < 20 mmHg}>-

'33

Page 137: Arterial Blood Gases Made Easy - I. Hennessey, A. Japp (Churchill Livingstone, 2007) WW

Index

N.B. PlJg.. 'tI.mbt'rs in ilJlIi<' dmo1rmuti'Tw/ ill Qur,;fwn 6' A"~'r !>'retlon

A-.'\ grndient 56, 106 7, 733acid-b.1.SC balannp 26-35

blood pH, nonn"l 20impo,t,mce 27mainlt'lwnce 29normaiJO. 31. 106-7, 1J1. 133stalu§5S

interp~l,llion54...-rulso pH

add--b<l!Pl" balanrf' d.:.orders 3(>-41acid-h....:.c disturbaT1('l."S 30-5

COffip<-'Itsated 32-3mixPd34-5.41uncomlX"ll::ioIted 30, J 1

.lC.d-base IlO<Jl<:lKraD'l 34,.\53Cldaemia. _'t>re 86-7, 12.1aridity 26ad.-lOSJS Tl

a<:id ba5C' status 55d..finibon 26,:lOsu a/$.O l1Ietabolj.c a<;iJOI>llI; mixed

addosis; nospiratoryaodosis

ilLut~ respirlllory distress "}'lldrome98- 9, J29

,..-tn.·n.d insuffictt:ncy 94-5, 127

alkalinity 26alkalosis 21

acid-ba!\f' "',lIUll 55definition 2", 1OSt'I' ulsn metabolic illkaJosis;

respiT.ltoryalkaloW;AIlPn lest. modified 11,47ah·eol.1r-c<,pilJary mPmbrilrle"

parli'll pn.'~uresalSah'eolar g.t."ll"l.UiltiOn. simplifit.'<I56alveolar \-ef\lil.ltlon 6, 7

ilnd /'110:1.12-16an"""ma 1i2-3. 121illllill)'Sb 1

anatomy. n:spiratory 1:!onion ~pp 37

in lIlPt"bobc acid05is 39, 5-1, 80-7.90 1,125

severl' 88 9.98-9,721, 124,119m salicyl.ul' poe>oning 91...J, 116

atll'nal blood gas (ABC) anal)'!'iscommon valu<!l> 'iO 1..ontr...indicationo: 43in cl,al;l_1S 48.-19inli>Tp....--l.ltion 52-5In practi~ (C'"~1itudi....) 59-108m t:realmf'nl gUldaoct"1

monilonng 49w;t'~ for 48-95« "/:;(1 urnplin&

....tf'l",al blood sampl~-M

Page 138: Arterial Blood Gases Made Easy - I. Hennessey, A. Japp (Churchill Livingstone, 2007) WW

INDEX

aspiration pneumoniil 70-1, 115aspirin (Sil.I~btl.')poisorung J.l,

92-J,126assessment of illness S('\tl'rity, ABC

in 48l'~amples 49

iBthm..ll,life.-threat~ng 76--7, WIatmosphenc pres.o;ure 5

base excess (51::)allSeiSment of illness se..'erity 48commOil ,"alul"S 51in metabolic acidom 36

bicaroonate(~)actu,d (plasma) (HCo,.act) 51~tandard (HCO)o'>t)51

bkarlxmiltc (HCO:J) balanceIn add-base balance 28, 29tr'I chronic hypen:apnia 22ind~is4B

in hypt'l"'Cl\lilatiofl24in metabolic ilcidosis 36In mebbohc .llkalosis 38in respiIilIOry impairmt'Tlt 23, 25

blood sampleshandling 46venous ll$ arterial blood 44, 101--5,

J3lbuffers 26

caldum concentration, plasmilIOf\ised (iCa)

common values 51low 78-9, 98-9,119, 129

carbon dw,xidc (CO~effect of shunting 15elimination 6

and alveolar ventilation 13in pulmorulry gas~ 4

"6

carbotl dio~departial prt'Slrurt'

(PCOz)5in air 5in arterial blood (POlWJ.l 5

common values 50III diagnosis 48IDCrcased 6Ul metabohc alkalosis 38in mixed acidoW 41r'l'a'ptocs 6, 7in respiratOf)' addosi~!

alkalosis 40in re6piratory irnpairmcnl20. 22and VIQ mismatch 14

common values 50and H' ;on remo.:al2Bin rt'5J>iraloX)' iJnpainnerlt 23, 25

carboft monoxide poisoning 80-1, 110cardiac arrest %-7, 128chloride (CI") ions

in met.moltc alkalosis 38pIa5ma Jeoveis. oommon values for: 31

chronic ob!;tructive pulmonarydISeaSe 66-9, 72-5,lJ3-14, rt6-17

CI ion~ sef' chloride (en iOllScompensation '19,32-3

full ~ partial 32, 33In metabolic alkalosis 38predicting 34respiratory tIS metabolic 54

COIltraindicat;om to ABC anal}"sis43

diabetes rnelhluistype 1 86-7. 113type 2 60-1, J10

diabettc ketoacidosis 32. 86-7, J2Jdiagnosis. ABC in 46, 49disordeP.l of l\Cid-base balance 36-t1

disorders of g3S exchange 18--25dJuretic drugs 38

Page 139: Arterial Blood Gases Made Easy - I. Hennessey, A. Japp (Churchill Livingstone, 2007) WW

e1ec:trol)'be rq>laCl'.ment therapy IJl)eodotracheal intubation 16, 17

exh.a~gascomposition during respiration

cycle 13effec:t of shunting 15

fan" masks, oxygen deli'.(>I')' 16, 1;FiO" s« o.w.ygen concentration in

,nspired airnuid replaC'm'Ol.'nt therapy 130

gas exchange disorders 18-25sununary2S

g.u exchangE', pulmonaryassessment 52-3basics 4-17defLnition 4

glUCO!lt', fa."iting. plasma ~e1s,(OlllU'lOn values for 51

H' ions s« hydrogen tons (W)hilemoglobin (Hb) plasma

ronct'nlration 8rommon values 51

haemoglobin oxygen saturation(SOJ8-11

in arterial bkx>d (~8, SOfal5ely high 81, 120and V/Q mismatch 14

common values 50lncrelO5e with tr.msfusion/lron

supplements 82-3, 121HeOl ba\:lnce U'l bicarl:>onate

(HCO;J balance

INDl'X

hydrogen ions (H')rommon valUl"'li 50generation 28in pH ZJ~~211

hypercapnia 6aS5eSSment of 5Il?'-"erily 52chronic t'S ocutt! 7clinical signs 23in obesity noin respiratory impilIrmcnt 22

hypen::hlQr.lcmic itd~s 90-1,125hyperventilation 12, 14,24,25,58-9,

109in adrenallnsuffidcncy 94-5,127causes 24in diabetic kdoacidOSlS 86--7, 123diagnosis 48in mesenteric ischaemia 84--5,122and metabolic acidosis 36in methanol poisoning 88--9, 124p;ychogcnic:M, 78--9, 11.'Jin renal tubular aodOl>is 90-1.

l25in ~\icylatepoOwning 92-3, 126

hyponatf;W'mja~, 127hypme.aemia 96-7, 118

1ll anaMtia 82-3asst'$SIr\eJlt of $t'\"erily 52definition 18~ 10 :maemia 127~ere80-l, 120

h:.poxia 18, 19hypm.ic <'Irivc 7, 113

in re;piratory im~innent22

impaired o:l(}'genatiOCl 18inadeq\1ale uxygenatiOn 18Inspired air (inhaled gas)

composition during respirahoncycle 13

Page 140: Arterial Blood Gases Made Easy - I. Hennessey, A. Japp (Churchill Livingstone, 2007) WW

INDEX

eff~lofshunhng ISoxygen concentraliOll (FK.l.J) 16

ml:et'pretabofl of ABGs 52-5

K- ions 5« potassium (K·) !oRo;

kidneysand mo..'tiIbohc alkalosis 381\.'ruI1 pH maintenance 28, 29renallub\llar acidosis, type 1

90--7,125Kus.sm;I.u1'!> respiration 36

ladnte. C(IfJUnOfl values for 51lactic lIC1dosis 37

III iIoCUle paf\C1'e'antis 98-9,11"In cardiac arrnt 96-7, 128itl mescnlt>rlc ischaemia 8~';. 122

mC"enlcric isoehaemia 84--5, IIImd:,lbolic acidosIS 30, 36--7

in aC\ll~pilrlCrt"ltilis 98-9. 129in adrmallll5utficioency 9+-5.717anicon gap In 39asst'SSID('flt of .wveritr 54definition 36in diabetic keloacidosis 86-7, 123d,agnosili 4Bhyperventilation in 24in meseoto"!ric is<haemia 84-5, IIIIn methanol poisoning 88---9, 114mixe..i with respiratory acidosis 41miX('(! with respiratory alkalosis

7O-J,92-3, JJS. 126In renal tubular addos.lS 90-1. 125m sil.ht:ylate poisoning 92~3. 126

metabolic alkalosis 30. 38-9in carbon lIlOIlOl<ide poisoning

80-1, 120

".

causes 39definition J8d'illgllOSlS 48in pylonc steTlosis 130with vomiting 100-3.130-1

metabolic pH mainte:runce 28, '19methanol poisoning 88-9, J24lTllXed i1od<l6i!; 41modifioo. Allen lest 44, 47

Nil.' ions Ii!« 5Qdium (Nj,°) >OOS

nJ~1 prongs 16, 17

obtsity 60-1, 1]0optold lonc:'ly 64-5. 112olC)'g.:n 10::)

Cllncefltrahoo in inspired air(FiOJ 16

deli>erydeo.·k516-17effl'd of shunting 15in pulmonary gas exchange 4_ also supp&emlmtal O>o:) gen

oxygen partial pre5SUIl:' (PO.z) 5in air 5In arterial blood (PdoJ 5

iilfId ah-ro1ar ventilation 12-161:common values SOin diagnosis 48III lmxed aodOSIS 41in oxyhllemoglobill

dissociation 10-11

-'in respiratn'y unpairmetlt20common vllues 50in oxyhaemoglobi.n dis'iOCiiltiOll

10--11in respiratory impainnenl 25

o")'gemrion 8-11and ah"eOl.1r vmtilatim> lJimpaired ~~ in..clequale 18

Page 141: Arterial Blood Gases Made Easy - I. Hennessey, A. Japp (Churchill Livingstone, 2007) WW

and oxygen coocentnltion 1f\

inspired air (FiOiI16oxyhaemoglobin dis5ocialion curve

10-11in respiratory impairment 20

PlfCO:! s« carbon dIoxide pam",1pl1'S9ure, in arterialblood

pancreatitis, acute 98-9, J29P.tOz Sn! oxygen. partial pressure, in

artedal bloodp.lrtW ~\tres5

sn /il$o carbon dioxide parti.1.1pressun>; oxygenpartial pnossure

Pcoz~arbon dioxide partial".,.,."~

pHblood

rommon values 50in diagnosis 48IlOrmal26in respiratory impairment 2J

dl!!inition ?J~le27

_ al$() acid-baSt' balancepickwidian syndrome ItOpneum.orua, aspiration 70-1.115pOz!# oxygen partial pressurepotassium (K') ions

in metabolic alkalosis 38plasm", levels, common values fO!"

51psychogenic hyperventiiatiOfl 24,

78-9,119pu1mQnlIry~lism 62-3, 106-7,

111,13Jpulmonary gas exchange

us smnent 52-3bilSics 4--17definition 4

INDEX

pulge ox.imetry 8, 59, 109pylork stCl'l(tiis 102-3, 131

radial artery sampling 44-5local atllH':5l1wtic 43wrist position 45

renal mechanismsin metabolic alkalosis 38pH maintenaru:e 28, 29

renal tubular addaMs, type I 90-7,125

respiratory acidosis 30, 40in chronk obslmctive pulmona.f)'~ 72-5, It4,1J6­17

chronic us acute 40COOlpeOS3tedJ2,33mixed with metabolic acic:b;is

"in opkrid toxicity 64-5, 112in I)"e' 2 diabetes 6O--J, no

resp;ratory alkalosis 30, 40in anaemia 82-3,121in <aspiration pneumonia 70-1, 1lScompen5a.tOl)' J2mixed with metabolic acidosis

7a-I,llSin psychogenic hyperventilation

'"in pulmonary embolism 62-3, 111uncompensated 58-9,]09

n:spiralory cen.tIe 6respiratory compensaltonsn

compensationrespiratory impairment (RI)

diagnosis 48s«.Iso type J respiratOl)'

impairment; type 2respiratory impairment

NSpiratory s)"5lem, pH maintenanrein 28, 29

R.l s« respiratory impairment

139

Page 142: Arterial Blood Gases Made Easy - I. Hennessey, A. Japp (Churchill Livingstone, 2007) WW

INDEX

!\alicyl.ale (aspmJ') poi9:lflil'lS 34,91-3,126

sampl~41 7contl'1lirldicarioM 4Jpuoctun" site!;.u. 4Jrolilia1artef}' 43. 44-5lM-fnnl': samptin,; 0\2-3"her samplinR.Mjtedmique

equiplJU.'1I1 42local m"l<--.,;;thPti<' 4.11'I.........ni k~pil'lg 42wrist poo;ition 45

vcnou.<; \'5 arterial bl\w 44

s.hunting 14, 15o;odium (Na·) ions

~n metabolic .....iJ06is 94-5. 12711\ rnetaoolJc ;Uk.l\oo<~ 38plasma ~'C15,. common value;; foe

51~truM' 70-1, 115:,uppWmenlal oxy~ n j

and AUG anaI~ Ib(aution with 73. 75. "r., 117in respiJatory impairment 20, 22type 1 R159, 67. 69, 109. 113, 114

t r('31 mMlt guidance!lJIl,mi toring.,ABC m49

lyPf! 1 respiratory impainnwt 20-1,25.~9, 109

140

1II.lwte pancreatitis ~8-9, 129III aspiration pncumwu., 70-1.

115~...-.~mmt of sevcnly 20. 21In asthma 76-7.118cau.5otS 21in ctuoruc obstructive pulJJ'\Ol\3.IY

d.isl'ase 66-7.]13 -

markers 21III pllhnnnary embobsm 62-3. IIIlrPatmmt 20

typ'" 2 respiratory in'p.-ljrnloPnt21. 25olcule-on-d\WlUC 74-.';. 117

CRUseS 23

chrorlll: 25l.IS ,\(111(' 2.1, 40

III chronic obstructive pUhllQ1li\ry •

disease 68 9, 72...."0. 114,n6-17

in opioid lQx)oty 64--,.';,112in pylon<: ,.h....1I06i.<; 1JOin type 2 Jiabt-If'"i 60-1. JJOWIth n.-niling 100-3. JJO....1

Vf'1lOUS blood sampk."S 41

ill3dvertmt 104-"0, 1.12ventilationlp.·rfusion (V IlJ)

mismatch 14illn::.piriltory impaimu.·(,t10

votluting 100-3, 13U-----Jand .>cid-base bul.llKX':lRin pyloric 5t~lS 1011-1. 131

Page 143: Arterial Blood Gases Made Easy - I. Hennessey, A. Japp (Churchill Livingstone, 2007) WW

EJqlloIno"'" ""rand how ......... OIl ABODoIoiIs .. tho '.............'0 01 abnanoaI;tyIncludrts Q unique ftow.chott for iflll p'elng resuIb25 purpclt 'Nti"'"' dtnicol em. scenorlos put the ABGinto pradice,

ThIs pocket guide IS flS8f'tticJ reading for 011 tho~ whorequire greater confidence in performing or interpretingABGs and is ideally suiled for medicol students, juniordoctors, specialist nurses ond nUrH proclilioners.

URCHILLL1V STONij

_N'"ww....ebe1'ierbes